Jump to content

Wikipedia:Reference desk/Science

From Wikipedia, the free encyclopedia

This is an old revision of this page, as edited by TreeSmiler (talk | contribs) at 02:17, 3 February 2008 (→‎Compact fluorescent lamps and flicker?: we have an article). The present address (URL) is a permanent link to this revision, which may differ significantly from the current revision.

Welcome to the science section
of the Wikipedia reference desk.
Select a section:
  • [[:|{{{1}}}]]
Want a faster answer?

Main page: Help searching Wikipedia

   

How can I get my question answered?

  • Select the section of the desk that best fits the general topic of your question (see the navigation column to the right).
  • Post your question to only one section, providing a short header that gives the topic of your question.
  • Type '~~~~' (that is, four tilde characters) at the end – this signs and dates your contribution so we know who wrote what and when.
  • Don't post personal contact information – it will be removed. Any answers will be provided here.
  • Please be as specific as possible, and include all relevant context – the usefulness of answers may depend on the context.
  • Note:
    • We don't answer (and may remove) questions that require medical diagnosis or legal advice.
    • We don't answer requests for opinions, predictions or debate.
    • We don't do your homework for you, though we'll help you past the stuck point.
    • We don't conduct original research or provide a free source of ideas, but we'll help you find information you need.



How do I answer a question?

Main page: Wikipedia:Reference desk/Guidelines

  • The best answers address the question directly, and back up facts with wikilinks and links to sources. Do not edit others' comments and do not give any medical or legal advice.
See also:


January 27

Summer Solstice

In the eleventh century what was the date for the summer solstice? Is it correct that the summer solstice swings between the dates of June 20 and June 24 over thousands of year? When (century) was the summer solstice June 24 (Midsummer's Day)? --Doug talk 00:15, 27 January 2008 (UTC)[reply]

see this calculator. Remember to convert the Gregorian result to a Julian date. - Nunh-huh 02:12, 27 January 2008 (UTC)[reply]

Cross breeding of rats and squirrels

Is it possible for rats and squirrels to crossbreed? I know they are different species, but there are cases of different species breeding (the mule being the most obvious). Also, in general, what are the requirements for species being able to interbreed? --Evan Seeds (talk)(contrib.) 03:28, 27 January 2008 (UTC)[reply]

The more closely the species are related, the more likelihood of success. If the species have recently diverged, it's more likely that they have a similar number of chromosomes, and it's more likely that their offspring won't suffer from a fatal lack (or surfeit) of genetic material. Horses have 32 pairs of chromosomes, while donkeys have 31 pairs, and the species have recently diverged: they both belong to the same genus. Rats have 20 pairs of chromosomes, while squirrels have 27 pairs, and have diverged far more: they both belong to the same order (Rodentia), but different families as well as different genuses. - Nunh-huh 03:43, 27 January 2008 (UTC)[reply]
Furthermore, even within the family Muridae the diversity is astonishing. Quoting from Nakamura et.al. :

the diploid chromosome numbers (of Muridae) range from 2n = 10 in Akodon species ... to 2n = 102 in Tympanoctomys barrerae.

Cheers, Dr_Dima. —Preceding comment was added at 15:53, 27 January 2008 (UTC)[reply]
I don't think that's too surprising. Rodents were the first mammals to evolve and have relatively short lives and breed a lot. As a consequence you'd expect a lot more evolutionary diversity there than in horses and the like, which are relatively recent and live relatively long lives. --24.147.69.31 (talk) 03:28, 28 January 2008 (UTC)[reply]
Strictly speaking, "Rodents were the first mammals to evolve" is nonsense: all mammals are equally descended from "the first mammals to evolve". I assume .31 means that the first mammals resemble today's rodents more than they resemble other orders. —Tamfang (talk) 23:16, 28 January 2008 (UTC)[reply]

Medical one way material

Im looking for the corrct english name for "medical one way material" (german: medizinisches Einwegmaterial). For instance most Scalpels, plastic stuff or medical syringes are made for one-way use only. I want to create this category in Commons. Is there an english article in Wikipedia about "medical one way material"? --84.137.47.124 (talk) 13:55, 27 January 2008 (UTC)[reply]

Probably "single use item" or "disposable item". -- Finlay McWalter | Talk 14:09, 27 January 2008 (UTC)[reply]
I think "disposable medical products" will do best. (most google-hits) Thanks! --84.137.47.124 (talk) 15:05, 27 January 2008 (UTC)[reply]
I think "single-use item" (with a hyphen, please) is much better. It conveys the fact that you aren't intended to use it more than once. An item that you use a few times and then discard, like a razor blade, is also "disposable". --Anonymous, 23:47 UTC, January 27, 2008.

Inhaler dream

Last night (in a dream) I was standing around in the ocean maybe 100 yards out (it was shallow) with friends I've never seen before. We had a gigantic inflatable raft that everyone was sitting on in deck chairs. One guy had asthma and kept coughing and using his inhaler. Someone splashed him from the water and his inhaler got a little wet. It started hissing a little.. he shrugged, capped it, and put it back in his pocket, then we all went in the water. The inhaler got totally wet and he pulled it out to use it I guess but foam was now pouring out of it. I shouted nooooooooo and dove for it in slow motion, grabbed it, and reached back to throw it, but someone grabbed my wrist and said hey dude not cool give it back to him. I shook off his hand and managed to toss it but he batted my hand and it only went a few yards. The asthmatic guy realized what was going on and dove in slow motion between me and where the inhaler hit the water. As soon as he came between me and it, there was a deep thump in the water and I could feel feet of water rushing past my legs. His eyes crossed in pain and an instant later, a mountain of boiling steam erupted from the ocean behind him. It was a few yards across and a lot higher than wide. He was blown forward and landed right in front of me, then I woke up. How much volume of gas is actually in one of these inhalers?! -froth —Preceding unsigned comment added by Froth (talkcontribs) 16:09, 27 January 2008 (UTC)[reply]

Metered dose inhalers usually contain a small amount of medication dissolved in a haloalkane. When full, the canister is completely filled with liquid haloalkane, and thus the volume of gas in the inhaler is about zero. When the inhaler is depleted, it will be filled with the haloalkane gas (at a much lower pressure) or air (depending on the valve mechanism), with a volume equal to the volume of the space in the inhaler bottle, I would guess around 25cc. Hopefully someone will come along shortly and be able to tell you the volume of gas at atmospheric pressure which is created from the volume of liquid in an inhaler, as that is probably what you are looking for, but until then, the answer to your question as stated would be somewhere between zero and the volume of the canister. Tuckerekcut (talk) 18:15, 27 January 2008 (UTC)[reply]
So no real explosive potential or catastrophic reactions with salt water? --f f r o t h 18:27, 27 January 2008 (UTC)[reply]
Well, Trifluoromonofluoroethane and Heptafluoropropane, two propellants used in inhalers, have vapor pressures of 70 and 44 psig, respectively, at room temperature. So a tiny amount of gas will build up inside the container until this pressure is reached. Suffice it to say I wouldn't want to puncture an aluminum can at 70psi anywhere near my face, but such a small volume of gas, and such a small mass of possible shrapnel is unlikely to do impressive damage, in my unproven and nearly anonymous opinion. Tuckerekcut (talk) 19:07, 27 January 2008 (UTC)[reply]
(Returning to a theme of a few weeks ago), in the process of "safing" inhalers so that I could recycle the aluminum canisters, I've punctured them with an automatic center punch; nothing exciting happens. (And yes, they still contained some of the propellant in liquid form so they were still in equilibrium at full pressure.) The canisters appear to be quite strong compared to the pressure involved, so a small puncture doesn't cause a catastrophic "tear-out" of the can.
Atlant (talk) 21:06, 27 January 2008 (UTC)[reply]
Once I managed to get an inhaler into the clothes dryer. The heat caused the gas to build up enough pressure to push the concave bottom of the canister into a convex bulge, so they aren't completely invincible. HYENASTE 03:11, 28 January 2008 (UTC)[reply]

Obviously, it can't be 100% as no money would be left for private industry, and obviously it can't be 0% as that would leave no money for public education, roads, airports, seaports, fire departments, police departments, etc.; thus providing no infrastructure for a growing economy. My question, then, is what is the ideal tax rate for maximum growth of the economy ? I realize this is a very difficult question to answer. I would expect that a chart correlating various net tax rates and long term growth rates in countries around the world could go a long way toward answering this question. Perhaps the net tax rate could be found by dividing the total of all taxes collected by national, provincial, and local governments in a country for a year by the annual national product. Do we have any such chart ? StuRat (talk) 17:28, 27 January 2008 (UTC)[reply]

Wouldn't the ideal rate depend on your personal or your nation's economic philosophy? Probably the lower it is, the higher the growth rate; the problem is (as you pointed out} providing public goods with a very low tax rate. Some of these things could theoretically be provided privately. I don't see why airports and seaports, for example, couldn't be owned and run by airlines/shipping lines (or a group of airlines/shipping lins owning shares). (It would probably raise prices, though...) Vultur (talk) 18:17, 27 January 2008 (UTC)[reply]
It definitely depends on the economic philosophy. As a libertarian, I think taxes should be as low as possible, and that most of the things on your list shouldn't be run by the government. Education should be privatized, and roads should be privately owned and maintained, as well as airports and seaports. (I also disagree with Vultur's prediction that this would raise prices. Competition in the market usually decreases prices in the long run.)
Also, I disagree with your (StuRat's) unquestioned use of "growth of the economy" as a figure of merit. A nation shouldn't be judged by its total size or wealth, but by the freedoms and quality of life enjoyed by its citizens. Growth isn't always good. (See VHEMT for another example of this philosophy that may surprising you.) Just some things to think about... —Keenan Pepper 19:16, 27 January 2008 (UTC)[reply]
I can't agree that lower tax rates are always better, which means 0% is ideal. That means no public education, which inevitably results in large portions of the population being ignorant and condemned to lives of poverty. This isn't just bad for them, it's bad for the economy of the country as well. StuRat (talk) 03:41, 28 January 2008 (UTC)[reply]
Tocqueville commented on the high degree of literacy, and appetite for books, among Americans a generation before the movement for public schooling (which is not the same thing as education) really got going. We might also observe the large portions of the population who are ignorant and condemned to poverty with public schooling. Like almost everything else on your list, education was done privately long before government stepped in and told us that it can't be done privately. —Tamfang (talk) 23:13, 28 January 2008 (UTC)[reply]
I believe that literacy rates are considerably higher in nations with compulsory public education than those without: See List of countries by literacy rate. Interestingly, communist and former communist countries score quite well on literacy, since, despite it's many other shortcomings, communism does promote compulsory public education (with the occasional exception, like Cambodia under the Khmer Rouge). Even poor nations, like Albania, manage a high rate of literacy due to compulsory public education. And, despite Tocqueville's comments, I believe US literacy rates were much lower at the time than now: [1]. Black illiteracy rates were around 80% in 1870 and dropped to under 2% by 1979, for example. StuRat (talk) 04:39, 29 January 2008 (UTC)[reply]
Wasn't it a crime to teach slaves to read? —Tamfang (talk) 16:34, 1 February 2008 (UTC)[reply]
0% works pretty well in Snow Crash and roads still get built and children still go to school --f f r o t h 19:55, 27 January 2008 (UTC)[reply]

Yes: Laffer curve. The details, e.g., progressivity and the needs of government such as war or a retiring population -- seem to be much more important than the average rate. MilesAgain (talk) 20:06, 27 January 2008 (UTC)[reply]

That seems to be the correct article, thanks. However, they only talk about it in theory, no actual numbers are provided. Certainly somebody could come up with some approximation of the ideal tax rate (10% or 90% ?). StuRat (talk) 03:41, 28 January 2008 (UTC)[reply]
Over at Laffer curve I found an excellent summary of any discussion on an ideal tas rate: here. --Ouro (blah blah) 14:55, 28 January 2008 (UTC)[reply]
Exactly; as I said, the percentage number is not as important as the other factors determining the ideal rate. Under some conditions, 1% may be ideal, but under others, 50% could be best. 20% could be good under progressive tax tables, but bad under a flat tax, all other things being equal. MilesAgain (talk) 19:32, 28 January 2008 (UTC)[reply]
The thing is, not all nations even collect tax. Some of the small oil-rich countries in the Middle East, for example, have the lowest tax rate in the world (if you mean tax the way I'm percieving it), and in fact give a reverse-tax system. I'm not sure of the details, though. Hope this helps. Thanks. ~AH1(TCU) 19:43, 1 February 2008 (UTC)[reply]

Mesozoic bird taxonomy

I'm having trouble finding out what family, if any, Paraprotopteryx is in. I'm working on the article for this genus now. It appears to be incertae sedis, but can someone confirm this? Vultur (talk) 18:18, 27 January 2008 (UTC)[reply]

Not much on google, is there? Found a wiki confirmation here[2] in the section Avian incertae sedis in the List of fossil birds. Julia Rossi (talk) 03:52, 28 January 2008 (UTC)[reply]

Fossils from mountains and islands

Sorry to ask two paleontology questions in succession, but is it true that mountains and small islands such as atolls have left no fossils, so we don't know what the Mesozoic mountain species were? Vultur (talk) 18:20, 27 January 2008 (UTC) —Preceding unsigned comment added by Vultur (talkcontribs) 18:19, 27 January 2008 (UTC)[reply]

Think about where there could have been rocks formed on a mountain. Most rocks are formed in hollows like lakes, rivers, oceans. Perhaps there are some fossils preserved under volcanic lava flows. Recent atolls are likely to have coral fossils, but mountain species are not likely to live on an atoll. Graeme Bartlett (talk) 21:20, 27 January 2008 (UTC)[reply]
Sorry, the atolls were a separate question. I read a book called "Life: The First Four Billion Years" where the author said that any islands in Panthalassa, when Pangaea was around, left no fossils or rocks at all - anything could have been there. I was asking how accurate that was. 98.199.17.94 (talk) 22:55, 27 January 2008 (UTC)[reply]
Isolated atolls on the ocean floor move around on the oceanic plate. Ususally oceanic crust is swallowed up in a subduction zone in a oceanic trench. This is broought down to great depths and usually melted to form magma. Fossils do not survive this treatement. It is possible for seamountains to be accreated to the side of a continental margin. But I don't know if any have done this from the Mesozoic or not. And if they did whether there were fossils. Graeme Bartlett (talk) 23:12, 27 January 2008 (UTC)[reply]
Also, just because there are mountains today doesn't nessecarily mean that they were there in the mesozoic era. The Rockies and Andes were just starting to come up, and some, like the Alps and Himalayas, weren't created until recently. The Himalayas, containing some of the tallest mountains on Earth, for exapmle, was created when India collided into Asia, which happened around 50 million years ago, after the mesozoic era. However, continents moving against oceanic plates can produce mountains and plateaus, as well as volcanoes. The Rockies and Andes are not moving against land, but rather the Pacific ocean, and the Deccan Traps have been around since the Cretaceous. Hope this helps. Thanks. ~AH1(TCU) 19:39, 1 February 2008 (UTC)[reply]

Big flashes of flame while cooking

When people are cooking something in a pan, probably mixing it around everything, and they make those big flashes of flame, what's going on? Where is the flame coming from? What's the purpose and how do I copy it? Does it have a name? ----Seans Potato Business 18:29, 27 January 2008 (UTC)[reply]

I think you mean Flambé. Hope this helps. --Dr Dima (talk) 18:40, 27 January 2008 (UTC)[reply]
In general add a liquid containing alot of alcohol and let the alcohol boil of - it then ignites - you need a gas stove to do this (or a match)
The alternative is flashes from very hot oil - such as can be obtained when stir frying - it's the same sort of thing - oil vapour is igniting - it needs to be very hot - and sometimes a lid is needed to put the fire out..77.86.108.68 (talk) 18:54, 27 January 2008 (UTC)[reply]
If Sean is thinking of stir frying, as in Chinese and Thai cooking, the flashes are definitely burning oil, not alcohol (i.e. not flambé). Alcohol (in cooking, at least) generally burns blue, slowly, and relatively cool, while oil flashing to vapor and burning is generally more yellow, quick, and hot. Also, flambé is a very deliberate, show-off technique, which is rarely done in the kitchen, rather right at the table in front of the consumer. (If you're going to add an expensive liqueur to a dish and then burn most of it right back off, you don't want to waste the effect, since the effect is all there is.) —Steve Summit (talk) 19:30, 27 January 2008 (UTC)[reply]
I'm thinking of the Chinese/Thai oil stuff. --Seans Potato Business 19:36, 27 January 2008 (UTC)[reply]
Get the oil very hot - then move the pan about like you are a proper chef maybe tilt it a bit - you really need gas - hopefully the oil will ignite.. if the oil is really hot so that it's spitting you may burn your eyebrows off etc. As I said before having a lid ready to put it out is a good idea. Also as far as I know it's not intentional and doesn't really add the the taste - just a hazard involved with cooking with very hot oil..83.100.183.193 (talk) 20:53, 27 January 2008 (UTC)[reply]
Southern Chinese cuisine often DO put rice wine (rich in alcohol) in stir-fries to aid the flame. --antilivedT | C | G 00:24, 30 January 2008 (UTC)[reply]
Flambé isn't just for show. Cooking that uses alcoholic beverages for deglazing the pan will often end with the flaring-off of the alcohol portion and the ignition is usually deliberate.
Atlant (talk) 21:11, 27 January 2008 (UTC)[reply]
It is relatively easy, using a flat cast-iron griddle on a gas stove, to get the oil hot enough that vapor forms, and then to ignite it at the edge of the pan with the stove flame. This is less common on electric ranges (no flame underneath). This would cause a grease fire across the entire pan; the stir frying method keeps things moving so that you only get flashes.
Flambe is similar in that there is a vapor that ignites, but the alcohol burns at a relatively low temperature and is quickly consumed. A large amount of oil in a pot, however, might continue to vaporize during burning, like a candle, such that the grease fire is fed continuously.
Turning the stove off would lessen the temperature of the oil, perhaps killing the supply of vapor. Covering with a damp cloth or lid would cut off the oxygen supply to similar effect. Throwing water on the fire would produce a bolus of steam directly under the flaming vapor, launching a fireball skyward with great destructive effect. Not advisable. Bckirkup (talk) 23:39, 2 February 2008 (UTC)[reply]

Danger of collecting blood samples for experiment

In such a case where one is collecting blood samples from the general population, there is surely a risk (to all parties concerned but I'm not addressing that right now) to the person collecting blood. Why then is not justified that they wear a mouthmask and safety glasses to protect their mucosal surfaces? How small is the risk? ----Seans Potato Business 19:34, 27 January 2008 (UTC)[reply]

As long as the vials of blood remain intact, there's no way for blood to get from the patient onto the mucosal surfaces of the phlebotomist. During the blood draw, blood is contained by the steel needle and then in the glass vials. In principle, it would be possible for a small amount of blood to be aerosolized if a vial were dropped and smashed, and for a small amount of that aerosol to settle on the mucosa of the phlebotomist. Similar aerosolization could occur if the phlebotomist were to fling blood-contaminated equipment about in the air. (The latter is an unlikely thing to have happen, and suggests a neglect of other basic safety precautions....)
Note that unless the blood is agitated to generate pathogen-laden aerosols, there is no risk from standing blood or blood droplets. (Clinicians and experimenters who handle pathogenic materials are – or ought to be – trained in techniques that minimize aerosol creation during the course of their tests.) Viruses, bacteria, and parasites won't evaporate, and won't travel through the air without help. Only small, non-infectious molecules (mostly water) will enter the air spontaneously by evaporation.
In short, aerosolization of blood during phlebotomy is very rare in the first place, and even where it does occur only small amounts of blood (and and correspondingly small load of pathogens) will be aerosolized. Of that pathogenic aerosolized material, only a vanishingly small fraction is likely to settle on a phlebotomist's mucosa. Many blood-borne pathogens – including many of the nasty ones like HIV and hepatitis C – are very ineffective at infection across healthy mucosa. Finally, in drawing blood from a general (superficially healthy) population for research purposes, the likelihood of the blood carrying any serious disease is quite small. The cost, inconvenience, and/or discomfort of masks and glasses aren't generally seen as a worthwhile tradeoff to defend against an extraordinarily tiny risk. Greater precautions may be taken in drawing blood from patients with known serious diseases or serious and unexplained symptoms. TenOfAllTrades(talk) 20:46, 27 January 2008 (UTC)[reply]
Okay, thanks for that explanation; but what about gloves and does it matter what material the gloves are made from? Are some more resistant to puncture than others? --Seans Potato Business 21:33, 27 January 2008 (UTC)[reply]
Generally the flimsy gloves used by clinicians aren't going to make a whit of difference if they're handling sharps incorrectly. Needles and broken glass are sharp; they'll easily go right through both nitrile and latex gloves. Individuals who work with biohazardous sharps are (or should be) trained in proper handling techniques. (Never recap needles; don't pick up broken glass with your hands; etc.) Intact gloves (nitrile and latex are the most common types) are all quite adequate for protecting individuals from bodily fluids. Different brands of gloves will have modest differences in stretchiness, comfort, fit, and durability. TenOfAllTrades(talk) 17:45, 28 January 2008 (UTC)[reply]
When I was a young 'un, I used to work in a lab that tested people for cystic fibrosis and one of my jobs was to collect the blood samples each day from the doctors and extract the DNA from it. I used to wear gloves, a lab coat and safety glasses, but no mouth mask. Only on one occasion did the blood aerosolize, and that was when I managed to accidentally launch a vial across the lab, smashing it and spraying three of us with blood. That was not a good day. Twice I had to extract DNA from blood from known HIV positive people (that was real bad luck, I thought, having HIV and cystic fibrosis) and I sure as hell concentrated on good safety technique those days. As a totally irrelevant aside, I also had to extract DNA from mouthwash/cheek swabs and we found that people who had eaten an apple just before almost always gave crappy DNA samples. We guessed it was something to do with the acid. I learned two important things from that job: make sure your blood vials are clipped firmly to the rotator, and don't eat an apple before a buccal swab. Good times. Rockpocket 08:02, 29 January 2008 (UTC)[reply]
My lab supervisor has had 1ml LN2 stored vials containing various pathogens explode in her hand (well, maybe not explode, but burst open and spray their miniscule contents about). She's always careful to hold them such that only her glove (and the water bath she's thawing it in) get hit should a vial explode. To my knowledge, she's never gotten infected...Though maybe that's considered cheating. Cells tend to be fairly pissy after coming out of the deep freeze. Someguy1221 (talk) 08:16, 29 January 2008 (UTC)[reply]

Glaciers

Are glaciers geological features? You might think this is for homework, but no. I did a science mind map two months ago and never figured out this question. Please reply. I would greatly appriecate it. 99.248.42.43 (talk) 20:19, 27 January 2008 (UTC)[reply]

The answer probably depends on your definitions. Most people would agree that a geologic feature needs a degree of permanence to it. Glaciers tend to stick around quite a while by human standards, but ice ages bear testament to the mutability of glaciers. Additionally, the current state of widespread glacier retreat may impact your decision. — Lomn 20:29, 27 January 2008 (UTC)[reply]
However, the Morraines left behind by glaciers are most definitely a feature of the geological record. SpinningSpark 23:50, 27 January 2008 (UTC)[reply]
If "geological feature" means something that people in the field of Geology study, then sure -- glaciers are definitely studied by geologists. But then glaciers are also studied by people in other fields, like Climatology, so maybe they are "climatological features"? Then again, Glaciology is the study of glaciers, which would make glaciers "glaciological features". Maybe the nature of something like a glacier depends on what framework you are working with. Just a thought. Pfly (talk) 03:53, 28 January 2008 (UTC)[reply]

Prehistoric love

I'd like to know more about this [3] archaeological finding. Thanks. --Taraborn (talk) 20:38, 27 January 2008 (UTC)[reply]

A bit more context such as where/when may help getting you a detailed answer.. if you can provide it. (Otherwise those two 'things' are skeletons - the inorganic remains of human beings in what is often called a 'dual burial').83.100.183.193 (talk) 20:58, 27 January 2008 (UTC)[reply]
[4], ,this one ? Mion (talk) 21:25, 27 January 2008 (UTC)[reply]
Yeah, that one. Thanks. --Taraborn (talk) 22:01, 27 January 2008 (UTC)[reply]
Double burials are not uncommon -- google for the term. The two individuals may be siblings, and other same-sex pairs may be lovers. In the end it is impossible to know, and we "read" a lot of our cultures into the cold objects. For a descriptive example in ficiton: Jean M. Auel used the archeological record of a double burial in her prehistoric novel The Plains of Passage. BrainyBabe (talk) 09:09, 31 January 2008 (UTC)[reply]

What do dogs search for?

When a dog walks around searching for exactly the right place to do its business, what is it really searching for? —Preceding unsigned comment added by 216.47.188.16 (talk) 21:28, 27 January 2008 (UTC)[reply]

As I understand it they are smelling around for scents and finding a place to 'mark their territory'. See here (http://www.hsus.org/pets/pet_care/our_pets_for_life_program/cat_behavior_tip_sheets/urinemarking_behavior.html) and here (http://home.howstuffworks.com/how-to-solve-dog-behavioral-problems10.htm) ny156uk (talk) 22:41, 27 January 2008 (UTC)[reply]
Does that mean if they do it in the wrong place that it may send the wrong signal and start a fight? —Preceding unsigned comment added by 71.100.3.254 (talk) 13:33, 28 January 2008 (UTC)[reply]
From what I know, yeah. If two dogs' territories overlap or border, or one starts marking an other dog's territory as his own, and the dogs perceive themselves (via this smell communication) as hostile, then they bark and growl at each other, and do who knows what else. There used to be a term for this olfactory communication, but I forgot it. --Ouro (blah blah) 14:50, 28 January 2008 (UTC)[reply]
That would be the pheromones. See this for an example of what could happen when an animal wees in the wrong place (and also what Rockpocket does for a living when he isn't killing time on the Ref Desks). Rockpocket 07:34, 29 January 2008 (UTC)[reply]

Calendar calculation

Taking into account the changes from the Julian calendar to the Gregorian calendar what would the date as we know today as June 16 be in the 14th century in Europe?--Doug talk 21:51, 27 January 2008 (UTC)[reply]

June 16th (Gregorian) = June 8th (Julian) in the 14th century (not including 1400, when it's June 7th), according to any number of online calendar converters. Algebraist 22:19, 27 January 2008 (UTC)[reply]

Thanks. Would that mean then that June 24 (Gregorian) = June 16th (Julian) in the 14th century? The Julian calendar was the one used in the 14th century, correct?--Doug talk 23:31, 27 January 2008 (UTC)[reply]

Yes and no. I'm assuming this is related to the Giovanni Boccaccio question at the Humanities desk. If you're interested in counting in exact 365- or 366-day periods between one event and another, then you'd say that exactly 695 such periods elapsed between 16 June 1313 and 24 June 2008. However, if Boccaccio was born on what was called "16 June" in 1313 (and we don't know that he was), then 16 June would still be his date of birth, not 24 June. That's because the Gregorian calendar was not applied retrospectively to 15 October 1582, the date it was introduced, and all Julian calendar dates prior to 15 October 1582 are still correct and valid. If Boccaccio had in fact been born on 24 June 1313 (the feast day of St John the Baptist) under the then-existing Julian calendar (and, again, there's no evidence for this), then we'd still say his date of birth was 24 June 1313, even though 695 full "years" wouldn't be finished until 2 July 2008. -- JackofOz (talk) 23:57, 27 January 2008 (UTC)[reply]

If you will also be dealing with dates at other times of year, note that the year itself may not be what you expect, as the year typically did not start on January 1 and the date when it did start varied from place to place. For example, the date we would call April 1, 1366, at the time in England would have been March 24, 1365, and the following day (the first of the new year) would have been March 25, 1366. Sometimes you will see the notation "March 24, 1365/66" used to remind you of this problem. --Anonymous, 00:00 UTC, January 28 (Gregorian), 2008.

"... the date we would call April 1, 1366 ..." - I disagree with that part of your answer, Anon, on two counts. (a) Regardless of whether the year was called 1365 or 1366, the day was called 24 March by the folks back then, and that is still the only correct way to refer to that day now. (b) You seem to be extrapolating by use of the proleptic Gregorian calendar, however this is a pointless exercise. Dates in the Julian calendar prior to 15 October 1582 were not adjusted, and shouldn't be, otherwise there'd be calendric chaos. There was a disjunct between the calendars, which is what the 10-day gap was all about. The Julian calendar applied right up till 14 October 1582 and the Gregorian commenced at midnight leading into the 15th. Essentially, the Gregorian calendar is as irrelevant to dates prior to the 1582 changeover as the Julian is irrelevant to dates after the changeover. If we were discussing a date between 1582 and 1752, when the UK adopted the Gregorian calendar, you'd be right to bring OS and NS dating into the equation, but we weren't. -- JackofOz (talk) 03:30, 29 January 2008 (UTC)[reply]
True. I should have said something like "that, extrapolating backwards by the calendar we now use, would be called." --Anon, 02:27 UTC (copyedited later), January 30 (still Gregorian), 2008.


January 28

Irreversible enzymes?

As a medical student rather well aware of chemistry, I have lately gotten a bit confused about the notion of enzymes catalyzing a reaction only in one direction. For example, hexokinase, an enzyme which catalyses the first step of glycolysis (turning glucose into glucose 6-phosphate), catalyzes the reaction in said direction. This has been referred to (by lecturers etc.) as an irreversible step, a step which is catalyzed in only one direction. The reaction can of course be reversed and is so in gluconeogenesis, where the reaction is catalyzed by another enzyme.

My questions regarding this are:

  1. What mechanism allows an enzyme to catalyse a reaction in only one direction?
  2. What effect will this have upon the balance of a reaction? While Kc should remain unchanged, adding e.g. hexokinase to a solution of glucose and glucose 6-phosphate should cause the balance to shift away from the equilibrium towards glucose 6-phosphate? OK, this is a bad example, since this step involves the hydrolysis of ATP, but suppose it didn't, or better up that my example was one which involved no other molecules but was otherwise the same, irreversible?

Thanks! ~Linus, 07:40, 28 January 2008 (UTC)

Strictly speaking all enzymes catalyze their specific reactions both ways; they do not shift the equilibrium. In the case of hexokinase (and some other enzymes), the reaction is called "irreversible" because the equilibrium clearly favors one side. Icek (talk) 14:22, 28 January 2008 (UTC)[reply]

Actually, the truth of the matter is that the equilibrium does not favor one side so clearly all the time. In the case of Glucose -> Glucose-6-phosphate, the hydrolysis of ATP provides a very favorable impetus for the forward reaction, which is why it is considered "irreversible". Nature couples ATP hydrolysis reactions with the reactions that need to go but don't go favorably enough.18.96.7.80 (talk) 16:17, 28 January 2008 (UTC)[reply]

Yes, ATP is the real secret escape from "catalysts don't change chemical equilibria." In a coupled reaction, you've added a new reactant and new products, and so the chemical equilibrium is changed as the chemical reaction is different. Someguy1221 (talk) 16:29, 28 January 2008 (UTC)[reply]

"Down's Syndrome" in animals

Does an analogous genetic disorder to Down's Syndrome exist in other species of animals? --Taraborn (talk) 02:48, 28 January 2008 (UTC)[reply]

Apparently not. However, this article says that by inserting human genes into lab rats, it may be possible to recreate Down's in other animals. [5]. bibliomaniac15 02:51, 28 January 2008 (UTC)[reply]
This website describes some very interesting mouse models of down syndrome, and suggestions on where to find papers on it. Someguy1221 (talk) 02:54, 28 January 2008 (UTC)[reply]
[6]. This states its in every population.its late.[7] [User:Mion|Mion]] (talk) 02:59, 28 January 2008 (UTC)[reply]
It's clearly referring to humans only, however. Someguy1221 (talk) 03:03, 28 January 2008 (UTC)[reply]
I think you're right Someguy,it's the extra 21st chromosome in humans that is named Down's Syndrome, now you can inject a artificial human something into a human/mouse to experiment or mimik effects, its still not the extra 21st chromosome in humans, i think similar errors happen in animals as well, but they have another name, so the answer is no the extra 21st chromosome in humans (down syndrome) is not present in animals. Mion (talk) 03:13, 28 January 2008 (UTC)[reply]
Some budgerigar breeders claim that the 'feather duster' mutation (which causes constant abnormal feather growth, deafness, blindness and mental retardation and is probably a result of too much inbreeding) is analogous to Down's Syndrome. I'm not sure if there has been any serious scientific research undertaken. --Kurt Shaped Box (talk) 07:35, 28 January 2008 (UTC)[reply]
Three Cases of Trisomy in the Mouse; A. B. Griffen; M. C. Bunker Proceedings of the National Academy of Sciences of the United States of America, Vol. 52, No. 5. (Nov. 15, 1964), pp. 1194-1198 states that trisomy which means a chromosom is present three times is also possible in mice.--Stone (talk) 07:52, 28 January 2008 (UTC)[reply]
I was under the belief that the white tiger in some of these pictures had Down's Syndrome but I believe this may be inaccurate and that it simply has a strange appearance due to huge amounts of inbreeding. Lanfear's Bane | t 12:15, 28 January 2008 (UTC)[reply]
Yeah, skull deformity !necessarily= Down's, even though it may look superficially similar. Interesting site though. It contradicts WP's own White tiger article with regards to the claimed 80% infant mortality rate, though... --Kurt Shaped Box (talk) 16:37, 28 January 2008 (UTC)[reply]

Is down's syndrome what most people refer to as "retarded"? 64.236.121.129 (talk) 17:09, 28 January 2008 (UTC)[reply]

Down's Syndrome is one potential cause of Mental retardation in humans. There are others. I don't know if it's considered polite to use the term 'retarded' (in a non-insulting manner) nowadays. --Kurt Shaped Box (talk) 17:19, 28 January 2008 (UTC)[reply]

Scientists have actually created a Down Syndrome mouse, it's called the Ts65Dn mouse. As you probably know Down's is caused by trisomy 21 in humans (pretty much all other trisomies are lethal, its seems one of those quirks of nature that human 21 is small enough, and has the right combination of genes, to permit survival if one has an extra copy.)

In other animals, though, genes get scrambled around to different chromosomes during the course of evolution. What the scientists did, was create a mouse with an extra piece of mouse chromosome 16 spliced to an extra piece of chromosome 17. This construct contains 104 of the 231 genes we find on human chromosome 21, and introducing that to mice they modeled the disorder. The result was not a perfect copy of Down's, but it produced mice that live and have the characteristic skeletal changes (a shorter, broader skull and jaw, for instance) and changes in brain structure that are seen in humans with the syndrome.

Since then, scientists have gone on to make mice with ever small numbers of genes, to try and understand exactly which of the 104 are responsible for the characteristics of Down's. Ts1Cje has extra copies of 81 of the genes found on human chromosome 21. Like Ts65Dn it models the human disease pretty well. Ts1Rhr contains just 33 extra genes, and these are the ones found in the human Down Syndrome Critical Region (a stretch on the long (q) arm of human chromosome 21 proposed to be responsible for some, if not all, of the features of Down syndrome). Surprisingly, this mouse didn't have the characteristic features of Downs. They concluded that the disorder is more complex than they had originally thought, and rather than being a result of an extra copy of few genes, the characteristics are a result of interactions between a number of genes. Rockpocket 07:25, 29 January 2008 (UTC)[reply]

Shit. Having written all that I note that someone posted a link to an article above explaining it in much more detail. Oh well. Rockpocket 07:29, 29 January 2008 (UTC)[reply]

Depth of ignition

At what depth and beyond will flammable material like clothing and paper inside a submarine hull ignite if the hull collapses? —Preceding unsigned comment added by 71.100.3.254 (talk) 08:00, 28 January 2008 (UTC)[reply]

I don't have any numbers for you. But, first: check the article on Self ignition. Next you want to measure how hot a gas will get when it's rapidly compressed, for that see Adiabatic compression. The final issue is that things will self ignite at a much lower temperature when the oxygen concentration is high. Now, I'm not sure that increasing the oxygen concentration by compression (not concentration really, but more like oxygen available near the item), is comparable to increasing it by having more oxygen, but it's got to have some effect. Ariel. (talk) 08:37, 28 January 2008 (UTC)[reply]
To the second part of your response, the answer is yes. In looking at the effective concentration of oxygen, the important term is the partial pressure of oxygen, not its relative fraction. Take ordinary room air (roughly 20% oxygen, 80% nitrogen) and compress it to a pressure of 5 atmospheres—that gives a partial pressure of oxygen of 5 atm * 20%: one atmosphere. Under those conditions, fires will burn essentially the same way they would under 100% oxygen at regular atmospheric pressure. (For similar reasons, deep-sea scuba divers use gas blends that contain reduced oxygen – heliox or trimix often contain 10% oxygen, for example – to reduce the partial pressure of oxygen that the divers breathe at extreme depths and reduce oxygen toxicity.) TenOfAllTrades(talk) 22:17, 28 January 2008 (UTC)[reply]
The question makes an untrue set of assumptions. Drawing on personal experience, submarine hulls do not get slowly, steadily smaller as they sink, yet keep their integrity and their shape, their internal volume shrinking, and air pressure increasing. Yes, there is some tensile yield in the hull, but for all practical purposes it will keep it's size and shape until some component gives way. At this point, lots of water comes spraying in, and two opposing mechanisms go to work. On the one hand, the incoming water rapidly fills the "people tank" as we called it, so the air is forced into a decreasing volume and the air pressure rises quickly. On the other hand, said air volume is also being kept "quenched" by the incoming spray of cold water. The gripping hand is that a submarine has very little that can actually burn. Possible fuels are limited to paper, fuel oils, lubricating oils, munitions, and the human body itself. Prior research - recordings made during the Thresher accident - showed very few noises that could have been called a diesel type fuel-oxygen detonation. The noises recorded were all the (expected) pops of breaking machinery and implosions of things that held gases but could not withstand the seapressure. Please note that I have never served on a submarine that suffered a hull implosion while I was there so I have to depend upon other witnesses and records. -SandyJax (talk) 19:57, 29 January 2008 (UTC)[reply]

Musical brains

Has any research been done into the structure and function of the brains of composers (as compared to the brains of the rest of us)? I ask because it struck me the other day just how amazing it is to be able to produce new tunes (almost at will in the case of some "tin-pan alley" composers). DuncanHill (talk) 10:29, 28 January 2008 (UTC)[reply]

The brains of composers are much more succulent than those of ordinary humans. Weasly (talk) 11:16, 28 January 2008 (UTC)[reply]
On the topic perhaps this link would be useful to you. Lanfear's Bane | t 12:05, 28 January 2008 (UTC)[reply]
Or this one (or this ) (must get round to reading this book myself) AndrewWTaylor (talk) 14:28, 28 January 2008 (UTC)[reply]

Phage titration / Identify bacteria

I have reading material written by a non-native English speaker and it says that "bacteria [can be] characterised by phage titration" (this is to confirm the integrity of an E. coli K12 DH5alpha culture). What are they trying to say? I think phage titration would be where you determine the number of phage per ml of a solution. ----Seans Potato Business 10:55, 28 January 2008 (UTC)[reply]

To test if the DH5alpha cells might have been over-grown by another strain of bacteria (or in some other way become less efficient as a host for the virus), you could test for the efficiency of infection using a virus stock. If all is well, you should see a high efficiency of infection in the test titration. If there is an apparent low rate of infection, then it might be time to grow up a fresh batch of the DH5alpha bacteria from a frozen stock. --JWSchmidt (talk) 04:06, 29 January 2008 (UTC)[reply]

Digital modulation transmission rates

I have a doubt about how can I manage the transmission rates on a system designed to allow multiple modulation schemes - for example, systems using the 802.16 standard. If the system uses BPSK I'll have a symbol rate equal to the data rate. If after a while I change the modulation, for example, to 16-QAM, the symbol rate will be 1/4 of the data rate, so, if the data rate remains unchanged, I'll not have a higher transmission rate, so, I wonder if those systems are operated by multiplying the data bit rate by the a constant dictated by the modulation in use (4 in 16-QAM, 6 in 64-QAM, etc.) in order to keep the symbol rate constant. Can you help me with that? Thanks in advance. 84.91.38.179 (talk) 10:58, 28 January 2008 (UTC)[reply]

I found something. It may answer your Question [8]--TreeSmiler (talk) 02:16, 29 January 2008 (UTC)[reply]
Thanks, it is just as I expected. If the baud rate changed after changing the modulation scheme it would not be possible to make a fair comparison between them. Thanks. —Preceding unsigned comment added by 217.129.241.169 (talk) 21:50, 29 January 2008 (UTC)[reply]

Classifying transformed bacteria (biohazard)

If I was handling, experimentally, a high concentration of a very immunogenic protein, would that pose a risk? I'm a uni student and as part of an assignment, I have to classify E. coli K12 bacteria that has been transformed with ppUL32 of the cytomegalovirus. The bacteria is class I, and the viral protein shouldn't be harmful in "normal" concentrations (the concentration at which it would accumulate in a CMV infection) but the protein is highly immunogenic (the most immunogenic of all CMV proteins). Thus, could the transformed bacteria be classed as class II? As a separate question, does the concentrated protein post an immunomodulatory threat? ----Seans Potato Business 12:27, 28 January 2008 (UTC)[reply]

Isn't the E. coli K12 used for educational purposes not the same "bad" E. coli found intestines. According to the escherichia coli article, E. coli K12 "have lost their ability to thrive in the intestine". -- MacAddct  1984 (talk • contribs) 20:33, 28 January 2008 (UTC)[reply]
Yes but it's important to take into account the danger posed by the genetic construct with which it was transformed. All E. coli K12 are not equal - indeed, some will be considered very hazardous and warrant level 4 containment, depending on the way in which they were modified (obviously strains modified in this way aren't used for normal educational purposes). ----Seans Potato Business 21:44, 28 January 2008 (UTC)[reply]
What kind of "immunomodulatory threat" are you imagining from this Cytomegalovirus protein? --JWSchmidt (talk) 03:49, 29 January 2008 (UTC)[reply]
I imagine some sort of harmful over-reaction to the concentrated protein, if it reaches a mucosal surface. ----Seans Potato Business 11:13, 29 January 2008 (UTC)[reply]
Um ... Wikipedia's probably not really the best place to be trusting for this sort of advice. (Not that we wouldn't know anything, it's just that were not *that* reliable in our answers - sort of the rationale behind the medical/legal advice rules: I wouldn't bet my life/health on the WP reference desk.) Your university should have a Health & Safety office which deals with research safety issues - I'd ask them. If for some reason your institution doesn't have one, ask around at the Health & Safety office of the nearest Big University. They should be able to give you answers, or if not, be able to talk to people (NIH, CDC, etc.) who would. For the most part, I've found (university) H&S people to be friendly and congenial. Don't worry about incurring their wrath - having a university researcher voluntarily consulting them is such a shock they tend to bend over backwards to help. (University researchers are notorious for ignoring safety regulations - by even considering the safety implications, you're doing far better than the average researcher.) BTW, if you haven't already, take a look at our Biosafety level article. -- 128.104.112.76 (talk) 22:52, 2 February 2008 (UTC)[reply]

What major would this fall under?

Lets say I want to build weapons, design new and different engines, cars and other vehicles. What college major would that fall under? 64.236.121.129 (talk) 17:08, 28 January 2008 (UTC)[reply]

Mechanical Engineering. Possibly aerospace engineering depending on the type of weapons and engines, but mechanical has that whole spectrum. 18.33.0.55 (talk) 17:26, 28 January 2008 (UTC)[reply]

British Rail Steam Locomotives

Some Tank engines used the suffix MT, what did the M stand for?90.198.148.196 (talk) 17:09, 28 January 2008 (UTC)[reply]

How was the suffix used? At LMS locomotive numbering and classification#LMS System it refers to MT meaning "mixed traffic"; that would be as part of a class designation rather than, for example, a wheel arrangement. --Anonymous, 23:40 UTC, January 28, 2008.

IMAX lamps -- as hot as the sun?

While waiting for an IMAX film to begin the other day, the trivia section in the beginning informed me that its lamps get as hot as the surface of the Sun. This claim is repeated on the web, e.g. here. Is this possible?

The Sun article says that the surface of the sun is over 5000 °C. At that temperature, isn't just about everything a gas? The article on the bulbs used in an IMAX theater, Xenon arc lamps, mentions that they use "fused quartz" electrodes. While I couldn't find anything on fused quartz, the Quartz article says that it has a melting point of about 1600 °C, well under the temperature on the surface of the sun.

Is their claim likely?

Thanks! — Sam

While you're quite correct that the solid components of the lamp would not survive at five-thousand-degree temperatures, the arc itself can get that hot. The arc is a hot plasma that only slowly conducts heat out to the electrodes and quartz lamp envelope. (This is why high-powered xenon arc lamps require water cooling—without it, the electrodes would heat to melting and the lamp would fail.) TenOfAllTrades(talk) 17:52, 28 January 2008 (UTC)[reply]
Thank you! — Sam —Preceding unsigned comment added by 63.138.152.238 (talk) 17:59, 28 January 2008 (UTC)[reply]
You might enjoy our article on Xenon arc lamps, the technology used in the IMAX projectors. And yes, the arc is very hot.
Atlant (talk) 23:46, 28 January 2008 (UTC)[reply]

How well would modern body armor stand up to musket fire?

Lets say, muskets used during the American Revolutionary War and rifled muskets used during the American Civil War. 64.236.121.129 (talk) 17:46, 28 January 2008 (UTC)[reply]

Body_armor#Performance_standards is relevant. There are many different levels of body armor. I would suspect that a typical musket of that era is comparable to say, a shotgun slug, in performance. Pretty much even the wimpy body armor should stop a large, slow-moving. not-pointed projectile. Speed and shape of the bullet make a large difference. Friday (talk) 17:52, 28 January 2008 (UTC)[reply]
Sounds about right. The soft non-jacketed lead will also aid energy dissipation. — Lomn 18:06, 28 January 2008 (UTC)[reply]

Do British men have higher pitched voices than American men?

If so, why? 64.236.121.129 (talk) 17:48, 28 January 2008 (UTC)[reply]

Apparently not. wikisource:The American Language/Chapter 29. "the American voice in general starts on a higher plane, is normally pitched higher than the British voice". Martinp23 19:04, 28 January 2008 (UTC)[reply]
I do think that British men are more likely to vary the pitch of their voices. DuncanHill (talk) 19:14, 28 January 2008 (UTC)[reply]
Compare Prince Charles and George Bush : ) Julia Rossi (talk) 02:59, 29 January 2008 (UTC)[reply]
Hardly what one might consider a representative statistical sample.  :) -- JackofOz (talk) 03:08, 29 January 2008 (UTC)[reply]
Perhaps you live in one of those countries, so you often hear one in person, but the other on TV/telephone, which might change the tone. – b_jonas 07:51, 30 January 2008 (UTC)[reply]

There may be a British tendency for the male voice to rise to a high pitch when bellowing orders in military drill, which is less likely among American soldiers, based on observation of soldiers drilling in London. I have also observed this when British men are yelling about anything. Edison (talk) 02:07, 31 January 2008 (UTC)[reply]

Species Identification

File:South African Insect.JPG
Who am I?

I was wondering if anyone could identify the species pictured in Image:South African Insect.JPG; the photo was taken in the Madikwe Game Reserve in South Africa which borders Botswana. —Preceding unsigned comment added by Guest9999 (talkcontribs)

It is a foaming grasshopper from the family Pyrgomorphidae. Genus Dictyophorus and probably D. spumans. Colourful South African grasshoppers tend to be poisonous. Take care. (online reference image) -- Lycaon (talk) 23:46, 28 January 2008 (UTC)[reply]
Thanks for the information. Guest9999 (talk) 01:31, 29 January 2008 (UTC)[reply]
That's a nice photo you took, Guest9999. -GTBacchus(talk) 00:01, 29 January 2008 (UTC)[reply]
Thanks. Guest9999 (talk) 01:32, 29 January 2008 (UTC)[reply]

β-mercaptoethanol uses

Along with smelling horrendous, I recall using β-mercaptoethanol as an ingredient for a DNA stain. I think it was when we did DNA sequencing. The article does not mention anything about its use in stains. Anyone have an idea? -- MacAddct  1984 (talk • contribs) 19:06, 28 January 2008 (UTC)[reply]

It is an antioxidant that prevents the formation of disulfide bonds in and between proteins. It is used in many biochemical buffers for that reason. Сасусlе 21:00, 31 January 2008 (UTC)[reply]


January 29

Alkaline drinking water

I saw this advertised on TV today: a machine that makes your drinking water more alkaline (to a pH of about 10) so that it supposedly neutralizes the harmful health effects of acidic soft drinks. Is there any scientific basis for this? Wouldn't the alkalinity actually make you sicker? --Anakata (talk) 01:01, 29 January 2008 (UTC)[reply]

I'd certainly have thought so. If it didn't give you acid burns to your throat on the way down, it would certainly have a good go at neutralising the acid in your stomach. Acidic soft drinks are certainly less acidic than your stomach acid, so if anything they would make your stomach less acidic. -mattbuck 01:23, 29 January 2008 (UTC)[reply]
Mild alkalinity isn't any worse for you than mild acidity is. Your stomach is already highly acidic, so a little acid or base either way isn't generally going to make any difference at all.
The harmful effects of the acidity in soft drinks are, I believe, the way they dissolve your teeth. So unless you took a quick drink of your allegedly-neutralizing tap water immediately after every soft drink you drank, I can't see this claim as having any validity. —Steve Summit (talk) 01:17, 29 January 2008 (UTC)[reply]
Have a good read of this before you buy anything. There is a heck of a lot of water-related quackery about. --Kurt Shaped Box (talk) 01:29, 29 January 2008 (UTC)[reply]
What you really want is this stuff. Someguy1221 (talk) 01:33, 29 January 2008 (UTC)[reply]
Heh. I remember seeing some new-age crackpot selling (expensive) little bottles of 'activated water concentrate' on one of those TV shopping channels a couple of years ago... --Kurt Shaped Box (talk) 01:45, 29 January 2008 (UTC)[reply]
Read about Phosphoric acid. It's pretty bad for you, and I'm not so sure that neutralizing it in the stomach will be all that effective. Ariel. (talk) 02:27, 29 January 2008 (UTC)[reply]
Phosphoric acid isn't "pretty bad". It is even an essential molecule for the cell - think of ATP hydrolysis, lots of phosphoric acid is created that way constantly. Nucleotidases hydrolyse phosphoesters in usual food, so, as far as I can see, phosphorus can only get into the body as phosphoric acid (or as some form of phosphate which rapidly attains equilibrium with the acid by taking up protons, and the other way around) in larger amounts. So virtually all the phosphorus that we need, including the phosphorus that ends as bone minerals, enters the body as phosphoric acid. The Wikipedia article mentions some studies which don't seem to contradict my assumption. Icek (talk) 21:06, 29 January 2008 (UTC)[reply]
Addendum: Nucleotides are not the only phosphoesters; there are also phosphorylated proteins, but I suspect that they're also largely hydrolysed in the stomach (are there digestive proteases which produce phosphorylated single amino acids from phosphorylated peptides?). Icek (talk) 21:11, 29 January 2008 (UTC)[reply]

Global Warming/Greenhouse Gases

The Wikipedia article on global warming shows percent ranges for greenhouse gases: 36-70% water vapor, 9-26% CO2, 4-9% methane, and 3-7% ozone. It seems to me the range for water vapor is impossible. If water vapor was under 58% it would be impossible to have 100% of greenhouse gases represented. I understand there are minor greenhouse gases that may be included, but nowhere near enough to make the percents come out to 100%. I would appreciate an explanation as I am using this data in a speech on climate change. Roy Mc (talk) 02:06, 29 January 2008 (UTC)[reply]

Like everything to do with global warming there are 1 billion sources saying 1 billion different things. It was (almost) universally accepted for a long time that water vapour contributed 95% - 98%. You would need to see check the sources from the article to figure this out.--155.144.251.120 (talk) 02:47, 29 January 2008 (UTC)[reply]
The problem is one of non-linearity and what does one mean by percentage of greenhouse gases (GHGs). If you remove all GHGs from the atmosphere except water vapor, then ~70% of the greenhouse effect would remain. In this sense water vapor is 70% of the effect. However, if you remove the water vapor from the atmosphere and leave the rest, then the remaining GHGs still absorb 64% of the affected radiation. In this sense the water vapor only contributed 36% of the total. The problem is that GHGs have overlapping absorption bands, so a portion of the radiation absorbed by water vapor can be absorbed by CO2, etc. and vice versa. Hence assigning a percentage to any particular gas is rather ill-defined. Dragons flight (talk) 10:16, 29 January 2008 (UTC)[reply]

Are these whales using tools?

See crows using "tools" here [9] and chimps here [10], I was wondering if humpback whales using bubbles to "net" herring [11], are also animals that are using tools? Julia Rossi (talk) 02:33, 29 January 2008 (UTC)[reply]

There's a species of shrimp which snaps its claws together to create a sound shock that stuns nearby shrimp. Is that a tool? The question here is, what do you define as a tool?18.96.7.80 (talk) 02:46, 29 January 2008 (UTC)[reply]

Clever little shrimp. I'm wondering what defines a tool and how much consciousness goes into making it for use. If a tool is a device used with conscious intent, technique's in there too... In the article about the apes scientists are excited because it's to do with the human evolution scale. What happens to that when you say more than chimps use tools? Julia Rossi (talk) 03:11, 29 January 2008 (UTC)[reply]
See sonoluminescence for the shrimp's thing Robinh (talk) 15:48, 29 January 2008 (UTC)[reply]
Animal cognition#Tool and weapon use.....I've seen people make the distinction between species that just use an available item as a tool and species that take an active role in making a tool or at least modifying a naturally occurring object so as to make it into a more efficient tool. These folks describe the whales as using a tool. --JWSchmidt (talk) 03:21, 29 January 2008 (UTC)[reply]
And then of course there's the question of what constitutes consciousness. It looks like a certain animal is "using" a "tool", but is it (a) innate/instinctive behavior, (b) accidentally discovered and remembered behavior, (c) learned behavior, or (d) deliberately contrived behavior? —Steve Summit (talk) 03:33, 29 January 2008 (UTC)[reply]
That makes it interesting – does this mean there's a difference between awareness of self and others, then self and things, and self and things and others? (If) a tool is one thing connecting a creature with another, a task or an outcome, is any animal known to use a "machine", then? like a simple two-part tool.—Preceding unsigned comment added by Julia Rossi (talkcontribs) 05:04, 29 January 2008 (UTC)[reply]
Personally I'd say no, that's not tool use. They aren't modifying anything; it's something their bodies can produce. Skunks aren't using tools when they spray someone. If we expand tool use to include physiological products of animals then we've destroyed the meaning of the word. Intentionality is necessary, but not sufficient, for something to be tool use. Whales using bubbles are not even in the running. Something much more on the edge would be a beaver dam, which is clearly a very complicated modification of existing resources for a very purposeful end, though they are not adaptable (it is clearly a very straightforward evolved instinct). --24.147.69.31 (talk) 14:42, 29 January 2008 (UTC)[reply]
Someone always chimes in with this, so I'll be the one today: whatever answer you arrive at will tell you more about the meaning of the word "tool" than the cognitive abilities of that animal. --Sean 14:48, 29 January 2008 (UTC)[reply]
Haha. Very enlightening. Thanks, all. Julia Rossi (talk) 02:55, 30 January 2008 (UTC)[reply]
Here's a recent article on how the brain thinks about tools. [12] --Sean 14:03, 30 January 2008 (UTC)[reply]

Does light have a finite distance it can cover?

My question is about the distance light can travel. Does it go on forever or does it "weaken" over distance eventually coming to a "stop" or "disappearing?" If I was in outter space and shone on of those million candle watt power flashlights toward a nearby star, will the light ever reach there? If not, what happens to it? —Preceding unsigned comment added by 216.154.19.46 (talk) 03:12, 29 January 2008 (UTC)[reply]

Currently accepted theories do not provide any mechanism for light traveling in a vacuum to weaken, stop, or disappear. (See tired light for a speculative idea that's not currently accepted -- not sure it's been definitively refuted but there's not much evidence for it and no theoretical basis to believe it).
However light does spread out, and become less intense that way. Even lasers (see diffraction). So while the photons from your flashlight will eventually get to the distance of the nearest star, I think there won't be enough of them in any reasonable amount of area for anyone to pick up a signal from you that way. --Trovatore (talk) 03:22, 29 January 2008 (UTC)[reply]
... and if the light is travelling for a very long distance then it becomes redshifted due to the metric expansion of space - although it has to be travelling for hundreds of millions of years for this to have a significant effect. But we can see light from distant quasars such as 3C 273 which has been travelling for thousands of millions of years. Gandalf61 (talk) 07:20, 29 January 2008 (UTC)[reply]
See Extinction (astronomy) for a discussion of how far light can go without bumping into too many bits of dust. --Sean 14:50, 29 January 2008 (UTC)[reply]
It might be werth mentioning that at the speed of light, time stands still, which is why there is no break down. See general relativity.--155.144.251.120 (talk) 22:24, 29 January 2008 (UTC)[reply]

I think that in space there probably is a tiny bit of gas, and debris drifting around so it will weaken eventually. My guess is that it will go a lot further than it will on Earth.KarateKid101 (talk) 10:32, 3 February 2008 (UTC)[reply]

the mute button

I can always hear things very slightly when it's "muted".. from TVs to my headphones. Is this a psychological phenomenon or real? :D\=< (talk) 03:46, 29 January 2008 (UTC)[reply]

I, too, hear this - indeed, I find that "mute" is louder than turning the volume control all the way to zero. DuncanHill (talk) 03:47, 29 January 2008 (UTC)[reply]
Yeah I agree. Why do they do this.. is there some property of resistors that when they're considering a rating for the "muted" resistor past a certain point it's too big/expensive/hot to go any more powerful? --:D\=< (talk) 03:52, 29 January 2008 (UTC)[reply]
There's no practical reason (e.g., "resistors") why mute has to be set to any particular volume, zero or not. Me, I tend to like the approach Sony sometime uses where "mute" is 20 dB down from the unmuted volume. But on our current Sony DSP-based receiver, "mute" is absolute; no sound/infinite attenuation. And in that receiver, it's all firmware-determined; no resistors are involved.
Atlant (talk) 11:58, 29 January 2008 (UTC)[reply]
My television has three settings that it toggles through on the mute button: Full Sound, Half Sound, No Sound. I assume they figure that some people want to just make the TV quieter and not get rid of all the sound. -- kainaw 15:12, 29 January 2008 (UTC)[reply]
You're not hallucinating. Mute is often accomplished by switching on a transistor somewhere in the pre-amplifier that shorts the audio to ground. That transistor, no matter how hard you turn it on, will have some slight resistance to it, and that will leave a tiny amount of audio to pass through to the amplifier. Also, you can get something like crosstalk from the proximity of parts of the circuit with audio on them to parts that are beyond the mute circuit. Power circuits like the final amplifier are made unhappy by having their load instantaneously switched in and out, so they don't just cut the output leads for mute, which would eliminate all sound. --Milkbreath (talk) 12:55, 30 January 2008 (UTC)[reply]
I seem to notice a similar phenomenon, but with video instead of audio. In some channels that I don't have but almost have, or with channels that I have but are so staticy and mute that they're nearly useless, I seem to notice the image move up and down, but occasionally especially with the ones that aren't just random static, it's possible to notice faint images of people or other images, but only with the channels that I do have is this effect pronouced, with the channels that I don't have the images are so barely noticeable that it's hard to tell if you're actually seeing them or hypnotised by the static, so the images are effectively useless, but with the ones that are pure random static you can't pick up any image at all. Thanks. ~AH1(TCU) 19:20, 1 February 2008 (UTC)[reply]
"They're heeee-eeere..."
Atlant (talk) 18:00, 2 February 2008 (UTC)[reply]

Gravity

I hope this question makes sense, since I am rather scientifically illiterate...but how big does something have to be before it has a gravitational pull? Is it relative, by which I mean will something very very small, like a mosquito, be affected by something relatively enormous, like a person? Adam Bishop (talk) 08:55, 29 January 2008 (UTC)[reply]

Anything with mass has "pull", as Gravitation notes in the intro. If you mean, how big does it have to be before you notice it, that depends on how sensitive your detector is. Clarityfiend (talk) 09:37, 29 January 2008 (UTC)[reply]
Newton's law of universal gravitation tell us that every object in the universe attracts every other object with a force that is is proportional to the product of their two masses and inversely proportional to the square of the distance between them. Einstein's theory of general relativity provides a more sophisticated and more precise theory of gravity, but in almost all circumstances it gives essentially the same results as Newton's law. So, yes, a person will gravitationally attract a mosquito - and, by Newton's third law, the mosquito attracts the person with the same force. The gravitational attraction between two such relatively small objects is very tiny, but it can be measured with sufficiently sensitive apparatus - see our article on the Cavendish experiment. Gandalf61 (talk) 09:37, 29 January 2008 (UTC)[reply]
As a follow-up, the reason that you're asking "how big" is that (relative to the other fundamental interactions), gravity is weak. It's 1036 times weaker than electromagnetic force, which is the other fundamental action responsible for most of our day-to-day experience. As that orders of magnitude article suggests, there's virtually nothing about 1036 for common sense to grasp on (best approximation I found: one atom to the number of atoms in all humans everywhere). Consequently, the masses in question also have to be far larger than what everyday experience grasps before the gravitational interaction becomes meaningful outside a laboratory environment. — Lomn 14:01, 29 January 2008 (UTC)[reply]

I should also note that size isn't a factor. It's mass. You can have a hot air baloon the size of Venus, but it won't have as much gravity as a neutron star, which is smaller in size. 64.236.121.129 (talk) 15:08, 29 January 2008 (UTC)[reply]

Cool, thanks everyone! Adam Bishop (talk) 17:13, 29 January 2008 (UTC)[reply]

Just to give a specific example, if a mosquito weighing 2 mg (about 1/14,000 ounce) is 1 meter (about 3'3") away from a person weighing 60 kg (132 pounds), then it is attracted to the person with a gravitational force of 8 femtonewtons, or about 1/35,000,000,000,000 of an ounce of force. And the person is attracted to the mosquito by the same force, because gravity works symmetrically. It really is a very weak force until you get planet-sized masses involved. --Anonymous, 02:40 UTC, January 30, 2008.
Lomn said gravity is 10^36 times weaker than the electromagnetic force. That is incorrect. Finding such a value requires dividing electric charge by mass. — Daniel 04:39, 30 January 2008 (UTC)[reply]
It's approximately correct; certainly it's correct enough for a discussion of why we don't find gravity to be practically relevant until we get to planet-sized masses. — Lomn 14:14, 30 January 2008 (UTC)[reply]

Expansion and contraction

I have an aluminum rod stuck in a brass pipe by nothing more than slight hand pressure. Now it is impossible to separate them. I've tried usign cold to shrink them and heat to eaxpand them but nothing. Is the cold shrinking the pipe more than the rod or is the heat expanding the rod more than the pipe or what is going on and what is the best way to separate them? —Preceding unsigned comment added by 71.100.3.254 (talk) 09:08, 29 January 2008 (UTC)[reply]

It's a bit iffy, since brass is an alloy and has a number of variations. Aliminum generally has a linear thermal expansion coefficient about 20% greater than brass at room temperature. So if you cool them down, the aluminum shaft will shrink about 20% faster than the pipe (again, varying depending on the brass alloy). You may need to really cool them down to make it loose enough, however, and I'd certainly suggest applying a lubricant at the same time. Someguy1221 (talk) 09:16, 29 January 2008 (UTC)[reply]
I did the cooling and the lubricant - but nothing. The rod will not budge and twists without turning whatsoever. Now I'm thinking put a little water in the pipe and cap it and then put the pipe in a fire... this should force both apart but maybe not or maybe the pipe will explode or the rod will become a missile. —Preceding unsigned comment added by 71.100.3.254 (talk) 09:26, 29 January 2008 (UTC)[reply]
Finally... what worked was a little colder temp and adding silicon spray to the oil. This seemed to free the grip the brass had on the aluminum which prevented turning and the cold and the relative softness of the aluminum seemed to work for axial motion. Eventually turning and pulling got the two apart with lots of scoring on the aluminum. —Preceding unsigned comment added by 71.100.3.254 (talk) 09:58, 29 January 2008 (UTC)[reply]

seeing through metal

would it be possible to create a sheet of metal so thin that you can see through it? --86.135.126.195 (talk) 10:27, 29 January 2008 (UTC)[reply]

Yeah, they coat glass in gold to block UV and you can still see through it sort of. Also just a few days ago I figured out that Pop Tart wrapper foil is so thin that it's transparent when you hold it up to your eye :D\=< (talk) 10:58, 29 January 2008 (UTC)[reply]
The pilots of the SR-71 Blackbird and Lockheed U-2 have helmets that feature a extremely thin layer of Gold on the visor, the visor is still perfectly clear and suitable for use, and the gold coating prevents the visor from freezing and misting up as it conducts heat and warms the visor ever so slightly. The astronauts that landed on the moon also used a similar gold plated visor on their spacesuits. We're still a long way from transparent Aluminium that is mentioned in Star Trek however. Nick (talk) 11:03, 29 January 2008 (UTC)[reply]
Our transparent aluminum article disagrees with you, but disappointingly provides no citation. Someguy1221 (talk) 11:08, 29 January 2008 (UTC)[reply]
(I'm pretty sure that the latest info in the transparent aluminum article could be sourced through New Scientist magazine; I think that's where I read about that. Perhaps some diligent Wikipedian could take a look... -- Atlant (talk) 12:03, 29 January 2008 (UTC))[reply]
[13] :-( Someguy1221 (talk) 12:21, 29 January 2008 (UTC)[reply]
That's surprising that New Scientist doesn't have 30 blaring articles about how you can make it in your basement. Tempshill (talk) 23:04, 29 January 2008 (UTC)[reply]
In this context see also Toronto's Royal Bank Plaza. --Anonymous, 02:50 UTC, January 30, 2008.
Closer to the idea of "just plain metal" (rather than a metalization layer), gold leaf is normally thin enough to see light through it.
Atlant (talk) 12:01, 29 January 2008 (UTC)[reply]

Gold is a good reflector for near infrared and longer-wavelength visible light, but not for blue light (hence its color). So it sounds strange that it is a good reflector for UV - are you sure, Froth? Or maybe the absorption of UV is relevant in the thin layers and is much higher than the combined absorption and reflection of visible light (but I doubt that)? Icek (talk) 20:54, 29 January 2008 (UTC)[reply]

Detailed info (with pics) how cable internet works

Hey everybody. I've had some trouble with my cable internet connection, and had a technician here today checking things out. Being the (huge) geek that I am, I became quite interested in how these things work, so I'm looking for some detailed info. I've read the articles at HowStuffWorks, but I'm rather looking for a "guided tour" of sorts, preferably with many images. I'm not very interested in the high-level things (i.e. anything above layer 1/2 in the OSI model), such as protocols or modulation, but rather what's beyond the jack in my wall - such as where the cable in the wall goes, what equipment is located there, and so on until we've reached the actual 'net. Thanks in advance :) Aeluwas (talk) 11:32, 29 January 2008 (UTC)[reply]

DOCSIS#Equipment would be a good place to start reading and work your way from the various articles provided there. It doesn't have pictures though. - Dammit (talk) 11:37, 29 January 2008 (UTC)[reply]
I have a book at home (which is titled something creative like Residential Broadband) that goes into this in great detail. If you like, I can probably find the exact title/author or perhaps you can find it on Amazon.
Atlant (talk) 12:05, 29 January 2008 (UTC)[reply]
Thanks guys. A book would be overkill, though (free ebooks aside)! Aeluwas (talk) 19:34, 29 January 2008 (UTC)[reply]
Well, if you eventually decide that you want a book, the one I was speaking of is titled Residential Broadband, Second Edition by George Abe, published by Cisco Press, ISBN 1-57870-177-5, $US50.00.
Atlant (talk) 00:13, 30 January 2008 (UTC)[reply]
Your library system might have a copy. Worth checking :) 130.88.140.119 (talk) 16:13, 1 February 2008 (UTC)[reply]

zero order hold

my question is that is zero orderhold a linear device?explain your answer in detailsSam kshitij (talk) 13:22, 29 January 2008 (UTC)[reply]

I'm not even going to try to read this, but we apparently have an article on it: Zero-order hold. Feel free to ask more questions if there's something you don't understand. Someguy1221 (talk) 13:29, 29 January 2008 (UTC)[reply]
Also see Linear system. Hint: the answer isn't simply yes or no. You have to explain what it means to be a linear device and under what assumptions a ZOH satisfies the requirements. —Keenan Pepper 13:38, 29 January 2008 (UTC)[reply]

Relation of ohms to watts in speakers

Hello, pretty noobish question here. Yesterday me and my friends bought two speakers from Savers for $3 a piece. They look like they have 8 inch speakers with a couple small tweeters. They have no text on them at all, no brand name or specifications. Using a multimeter, I found out that they are rated at 8 ohms which I understand is the standard for speakers of this size. I have a stereo in my basement (with a quite old Marantz preamp very similar to this one) which reads "185w" on the back. It is hooked up to two very worn out speakers, rated at 8 ohms. What I'm wondering is, does it matter a whole lot what the wattage of the preamp is as long as the resistance is correct? I'm basically wondering if pairing these new speakers with my old preamp will work, considering that they weren't made for each other but they are compatible in the sense of ohms. I don't want to blow these speakers I just got so I don't know if 185 watts is too much or what. Thanks a lot! NIRVANA2764 (talk) 15:30, 29 January 2008 (UTC)[reply]

Without going into technicalities I don't remember anymore, the amplifiers need something to work against, and they were designed for an 8 ohm load. You have an 8 ohm load. You're good. End of question 1. Question 2 is "Is 185w too much?" That's the rated power of the final stage power amps, and probably is too much for the old speakers you want to use. The solution? Don't run the amp at full power. In English, that means keep the volume turned down. Try to pump too much power thru those speakers and they will object. Loudly, if briefly. -SandyJax (talk) 22:14, 29 January 2008 (UTC)[reply]

Divergence of a species (speciation)

After much reading, I'm still not exactly sure on the mechanism of speciation. I understand the ways a population can get divided and be subject to different selective pressures. I also understand that it's populations that evolve, not the individual. However, doesn't it come to a point where individuals (or maybe "generations of families" is more appropriate) must evolve to make themselves incapable of producing fertile offspring? Maybe I just answered my own question... I guess when species don't evolve "together" and become incapable of interbreeding, that's natural selection at work? -- MacAddct  1984 (talk &#149; contribs) 15:37, 29 January 2008 (UTC)[reply]

Mutations are at the heart of evolution, and they do indeed occur in individuals. So in a certain colloquial sense I think you could say that individuals can evolve. I think when people say that "individuals don't evolve", they mean two things: First, individuals don't transform partway through their lives like on certain Star Trek episodes. Second, and more importantly, biologists define evolution not as speciation or as mutation but as changes over time in the frequency of given alleles in a population (I'm pretty sure that's right, even though it isn't quite how our Evolution article defines it [now that I think about it, it's pretty much saying the same thing]). So by definition evolution occurs in populations rather than in individuals. As far as speciation goes, I think you're on the right track. Two parts of one population diverge in their traits until they can't or don't interbreed. But if a single population changes together over time, that's still evolution (by the formal definition) even if it isn't speciation. --Allen (talk) 18:12, 29 January 2008 (UTC)[reply]
Oh, one more thing... evolution, whether it involves speciation or not, can be driven by genetic drift as well as by natural selection. --Allen (talk) 18:15, 29 January 2008 (UTC)[reply]
I guess my confusion is (although it may be an unintentional straw man) that a group of species must all evolve the same characteristics at the same time that would make them all fertile with each other, but different enough from the old niche. Even I feel like I'm misrepresenting something though... -- MacAddct  1984 (talk &#149; contribs) 19:13, 29 January 2008 (UTC)[reply]
Of course a large population can be split in two over time. The classical example for instance is the slow appearance of either a mountain range or a river which would keep two populations of a species separate. This prevents genetic material being exchanged between these populations.PvT (talk) 21:01, 29 January 2008 (UTC)[reply]
I recall reading something about two species of insects that only recently split off from each other and are almost genetically identical except that one reproduces at night and the other in the day (or something like that) so that they weren't separated by a physical obstacle but by a mutation that was able to persist. — Ƶ§œš¹ [aɪm ˈfɻɛ̃ⁿdˡi] 22:20, 29 January 2008 (UTC)[reply]
Well, it normally takes a good number of changes before two populations can't interbreed, so we're talking about thousands to hundreds of thousands of generations. That is usually enough time for the changes to be available throughout the population unless one of two things happens. First, if the two populations are isolated or mostly isolated from each other, then they can't exchange genes, and yes, this will cause speciation. In fact, that's the most common cause of speciation. However, in some rare cases, a species living in one area may separate into two groups, and then evolve apart from each other. For example, a mate selection trait may appear, and one part of the population will select mates with one particular trait, and another part will select mates without that trait, and eventually genetic drift and mutations will cause them to become separate species, despite coexisting (see Sympatric speciation). -- HiEv 04:48, 30 January 2008 (UTC)[reply]
Bear in mind also that speciation isn't necessarily all-or-nothing (though it usually is). I believe (though I haven't found a WP article on it) that there are cases where variation occurs across a wide geographical range, such that neighbouring varieties are mutually fertile, but the extreme forms are not fertile with each other. --ColinFine (talk) 00:07, 4 February 2008 (UTC)[reply]
Ring species. --Allen (talk) 03:33, 4 February 2008 (UTC)[reply]

Mystery metal

Since there appear to be some metal experts manning the reference desk (going by a few topics above) can anyone tell me what kind of metal that is extremely hard at room temperature - so hard in fact the it requires extra effort to be penetrated by a high speed steel drill bit yet melts into a blob similar to aluminum under an alcohol flame but with no slag. —Preceding unsigned comment added by 71.100.3.254 (talk) 16:50, 29 January 2008 (UTC)[reply]

Physical properties (like hardness and melting point) are related to chemical properties (strength of alloy crystal bonds, etc). Something that is very hard generally also has a relatively high melting point, because these properties are related. Do you have an example of this magical material? Some possibilities include (for some values of "room temperature") normal frozen water, or ice. Ice is very hard, yet has a pretty low melting point. I know, it's not a metal.
Also, does anyone know the flame temperature for "alcohol"? I assume ethanol is meant, but that's not in Gas burner. -SandyJax (talk) 20:33, 29 January 2008 (UTC)[reply]
Most alcohol stoves use Isopropanol, or a mixture of Methanol and Ethanol. The latter is more commonly known as Sterno. As for metals, you might want to look at Tin and Lead -- specifically, the mixture used in solder. The melting is about right, but the hardness...not so much. Also look for Gallium, which melts at skin temperature! --Mdwyer (talk) 22:04, 29 January 2008 (UTC)[reply]
Any of the fusible alloys should meet your melting requirement, although I'm not sure what any of their strengths are. Someguy1221 (talk) 22:14, 29 January 2008 (UTC)[reply]
A high speed drill would create temperatures in the vicinity of an alcohol-fueled flame rather rapidly, thus meting it and reducing drill-resistance. I am aware of some materials which become hard (though not very hard) when exposed to certain stresses, and yet are typically fluid at room temperature. Perhaps there is a material that fits the description with respect to physical forces (that is, a material which melts at the specified temp, yet becomes an oobleck-like fluid once it melts), but is not a metal. Tuckerekcut (talk) 22:32, 29 January 2008 (UTC)[reply]
An extremely hard metal is osmium, however that also has a very high melting point. Hardness and high metling point usually go hand in hand. But you may be able to have a mixture of say fine emery particles in solder. The drill will struggle, but the substance will melt. Safes may be built with metal with hard particles mixed in to make them hard to drill. Graeme Bartlett (talk) 01:54, 30 January 2008 (UTC)[reply]

The material reacted to the drill like glass or crystal in a brittle sort of way unlike steal which the high speed steel drill easily cuts through. I do not think there are any particles added but it is possible. The material was used simply to make end caps for threaded household lamp wiring tube. A center punch dented it but the drill required extra ordinary pressure to get it to go through. —Preceding unsigned comment added by 71.100.3.254 (talk) 08:22, 30 January 2008 (UTC)[reply]

Fascinating - I can't think of such a metal - those that would melt in an alcohol flame include tin/lead ie solder, maybe mercury alloys and possibly zinc (or aluminium) - none of these is very hard at all. Does the stuff break or dent when hit? how easy is it to - produce filings of with a file and how easy to saw through with a fine toothed saw ???87.102.77.153 (talk) 12:03, 30 January 2008 (UTC)[reply]
(also what is "threaded household lamp wiring tube" - is that an electrically conducting part?)87.102.77.153 (talk) 12:11, 30 January 2008 (UTC)[reply]
No it resists sawing and filing like it does drilling and dents but does not crack or break when hit with a center punch. Its the relatively low melting point that has me baffled as the threaded household lamp wiring tube is made of steel. The "threaded household lamp wiring tube" is the tubing used hold lamp parts together yet allow electrical wiring to be run through the tubing. The treads are on the outside and the inside is not threaded. All of the other end caps and connectors and threaded fittings used with it are either steel, brass or pot metal. This metal is very shiny and silvery after melting and more like polished aluminum than anything else. Maybe there are other tests I can do with acids and bases that will tell me what it is. —Preceding unsigned comment added by 71.100.162.155 (talk) 06:49, 31 January 2008 (UTC)[reply]
Right - a hollow tube with a screw thread on the outside I think .. and the material attatches to it. Sorry I can't think of such a hard low melting thing - and it's definately not zinc, lead, tin , solder etc?87.102.33.230 (talk) 08:04, 31 January 2008 (UTC)[reply]
Bismuth is low melting too, but I don't think as strong as you describe.
Questions.. after melting does the shinyness fade slowly to a more grey colour? does it burn in any way in a flame or show any rust/oxidations?87.102.33.230 (talk) 08:18, 31 January 2008 (UTC)[reply]
Also antimony is used in 'low friction allows' - could a slippery metal account for it;s difficulty to drill ?? a slippery antimony allow is my best guess87.102.33.230 (talk) —Preceding comment was added at 08:23, 31 January 2008 (UTC)[reply]

Rotten Coconuts

So I've been craving a coconut for a while now, but the last four I've bought from two different grocery stores have all been rotten. If I had to guess, I'd say it was because it's winter and it's out of season for coconut harvesters. Am I correct or am I just unlucky? (I live in Minnesota if that helps) --Ouzo (talk) 20:23, 29 January 2008 (UTC)[reply]

Since the coconut palm is a tropical plant, I'm not sure winter is a factor. So far I haven't found any specific references regarding that the coconut "season" might be. Maybe your grocery stores have had their coconuts lying about for too long. Is there a way you can ask for a fresh shipment from them? Or, there are several companies listed on this page which indicate that they deliver. --LarryMac | Talk 21:03, 29 January 2008 (UTC)[reply]
This coconut is always fresh. DuncanHill (talk) 22:54, 29 January 2008 (UTC)[reply]

A few weeks ago, a number of the coconuts found in the shops and markets in Israel were rotten with mould. Could there be some blight today affecting coconuts? Simonschaim (talk) 09:08, 30 January 2008 (UTC)[reply]

Commercial aspects using License Plate Identification software

Is there uses now in large shopping centers or business districts or industrical parks of a large city where the use of License Plate Identification software is used to track potential customers or repeat customers or criminal activities within these areas? Wikipedia article? —Preceding unsigned comment added by 66.255.74.101 (talk) 20:28, 29 January 2008 (UTC)[reply]

Yes, we have an article on Automatic number plate recognition. (EhJJ) 20:47, 29 January 2008 (UTC)[reply]

Unknown husk

Excuse me, what is this? It's one half of a egg shaped husk I found outside, made of dry reedy, wickery, balsa-woody material. It is extremely light, seems to have a seed pod inside it, and smells very nice, like the kind of thing you get in those baskets from the shops that sell the fancy soaps... What is it? Thanks SGGH speak! 23:42, 29 January 2008 (UTC)[reply]

Could be Mace (spice) the covering of a nutmeg. DuncanHill (talk) 00:06, 30 January 2008 (UTC)[reply]
Nice shirt! ;) Rockpocket 06:34, 30 January 2008 (UTC)[reply]
It does look like the seedpod in the mace piture SGGH speak! 08:58, 30 January 2008 (UTC)[reply]
Where did you find it? As in, which geographical region. − Twas Now ( talkcontribse-mail ) 02:33, 2 February 2008 (UTC)[reply]

January 30

dissapation of light

seriously how come if you were to put a candle in the middle of a dark place(room,ally hallway) it would not illuminate the entire area is it simply because the light is not powerfull enough or is it because it really dissapates,but i always thought that matter can't be destroyed just changed —Preceding unsigned comment added by Gotnse (talkcontribs) 00:01, 30 January 2008 (UTC)[reply]

Who says it doesn't illuminate the entire area? Try these experiments:
  1. Place a lit candle at one end of a dark hallway. Stand at the other. Can you see the candle flame? If so, photons from the candle flame are making it all the way down the hallway.
  2. Place a bicycle reflector on the wall at the other end of the hallway from the candle. Stand next to the candle. Can you see red glints from the reflector at the other end? If so, photons are making it down and back.
  3. Stand at the other end of the hall from the candle, with your back to the candle. With your eyes completely adjusted to the dark, stare intently at the (ideally white) wall. Have an assistant block the light from the candle at some point, without telling you. Can you see a change?
  4. Again at the other end of the hall from the candle, and with your eyes completely adjusted, see if you can see the shadow of your hand on the wall.
I'm not sure if numbers 2, 3, or 4 will work, but I can assure you that the candle does "illuminate" the whole room, even if it's not enough illumination to, say, read by.
See also this previous question. —Steve Summit (talk) 00:31, 30 January 2008 (UTC)[reply]

Well, it will only "illuminate" things it hits and bounces back off of into your eye. So although it looks like the light it dissipating its jut not bouncing off shit. —Preceding unsigned comment added by 69.159.193.94 (talk) 04:53, 30 January 2008 (UTC)[reply]

In re matter can't be destroyed just changed - light isn't matter. --Ouro (blah blah) 08:57, 30 January 2008 (UTC)[reply]
And besides, matter can be destroyed: e=mc² and all that. As far as the primary question, though "isn't powerful enough or dissipates" is a bad use of "or". The candle isn't powerful enough (for a specific application, such as reading a book) because its intensity dissipates as a function of the cube square of distance (surface area of a sphere). Other respondents will note that "because" isn't exactly the right word, but I hope it conveys that your two criteria are not exclusive and in fact are quite related. — Lomn 14:10, 30 January 2008 (UTC)[reply]
You mean square of distance, right? Algebraist 16:34, 30 January 2008 (UTC)[reply]
Oops, yes. Typed SA, thought volume. — Lomn 19:19, 30 January 2008 (UTC)[reply]
What's interesting is how the fourth power of distance is relevant in some cases and senses. If you have a diffuse reflector of a fixed size, and you're near the source of the light (e.g., a building being illuminated by your car's headlights), then the total light you see reflected by the object is inverse quartic. But since the apparent size of the object drops as the inverse square, the brightness falls off only quadratically as well. See also Lambert's cosine law, where cosines and distance interact in a similar fashion. --Tardis (talk) 17:28, 30 January 2008 (UTC)[reply]

Best way to destroy an old, (almost) non functional hard drive?

I replaced one of my drives today. The old drive was giving me lots of read/write errors and was making a grinding noise all the time and I was unable to securely erase the contents before I removed it. What's the best way to render it completely non-functional and/or make the contents non-recoverable to your average bin-raiding identity thief, given that there is quite a bit of personal information/private correspondence stored on it and it does occasionally still sort-of-work.

I'm not going to blast it with a shotgun and I don't have the facilities to melt the thing, before anyone suggests that... --Kurt Shaped Box (talk) 00:10, 30 January 2008 (UTC)[reply]

If it's in the middle of a head crash, you don't have all that much to worry about; why not just let it keep ruuning until it totally trashes itself? But if you like here are two other possibilities:
  • Take your Torx screwdriver, disassemble the drive, and turn the platters into ornaments or throw it all in the metal recycling bin at your local dump/landfill/tip.
  • Take your electric drill and about 3/4 of an inch out from the center of the spindle, drill a hole straight through the drive. If anyone attempts to run the drive, the hole will rip all the read-write heads off of their gimbals.
Other folks have suggested other creative means in past Reference Desk posts.
Atlant (talk) 00:20, 30 January 2008 (UTC)[reply]
I've found smashing it with a large hammer to be both effective and enjoyable. Someguy1221 (talk) 00:26, 30 January 2008 (UTC)[reply]
Like Someguy1221, I used a sledge hammer on an old drive from a school (containing both staff and pupil records). I can recommend this as a stress-relieving activity.dbfirs 16:08, 30 January 2008 (UTC)[reply]
after edcon
Open it up (using screwdriver, spanners, pliers etc), take out the platters,(made of aluminium) and
a)Put them through a bulk degmagnetizer that they use to have for magnetic tapes (could also try a hand held head demagnetizer).
Or, b) you could try putting the platters in the microwave oven (CAUTION: I dont know what gases might be given off so perform at your own risk).
Or, c) you could take out the platters and sand them down with emery cloth etc (esp using a sanding attachment to an electric drill). That should make 'em pretty hard to read!
Or,d) you could put the platters individually into your lathe, and cut off the magnetic coating with your tool.
Or,e) you could take the platters and just cut them into pieces with your hacksaw or tin snips.
Or f)you could just file across the platters with your big file.
Or g) maybe you could just torch the platters with your blowtorch (gas warning again)
Or.......--TreeSmiler (talk) 00:29, 30 January 2008 (UTC)[reply]
(ec) How resistant to heat is the magnetic stuff? Maybe bake it in a hot oven? DuncanHill (talk) 00:30, 30 January 2008 (UTC)[reply]
It will lose its magnetism beyond the Curie temperature but magnetism will then return when the magnetic material cools down. Whether the data will still be there is a very interesting question.--TreeSmiler (talk) 00:37, 30 January 2008 (UTC)[reply]
Definitely open it up. Supposedly those platters are quite pretty. (I've got a drive I've been meaning to do this with, containing tens of thousands of imperfectly-erased customer records, including credit card numbers 'n' stuff.) —Steve Summit (talk) 00:48, 30 January 2008 (UTC)[reply]
They are very flat and very reflective (if uncrashed). For people like me (who are easily amused), you can have a lot of fun just sticking two platters together and peeling them apart; because of their amazing flatness, that can be surprisingly hard to do.
If they're crashed, they can still be attractive but in a "surface ground" sort of way. But crashed disks can also be very dusty and the very fine dust can be as annoyingly hard to remove as toner.
By the way, most disk drives will still spin up and run, at least for a little while, once opened up. But if they're in the midst of crashing, please beware of high-speed flying ejecta (such as the heads!) from the disk drives.
Atlant (talk) 12:56, 30 January 2008 (UTC)[reply]

1. Smash it with a hammer a couple of time's 2. Drive to the nearest lake 3. Weigh it down and throw it as far as you can 4. ??? 5. PROFIT! 2. w —Preceding unsigned comment added by 69.159.193.94 (talk) 04:55, 30 January 2008 (UTC)[reply]

Put them on gravel and drive over them a few times? No car? ask a friend. Julia Rossi (talk) 07:20, 30 January 2008 (UTC)[reply]
Wouldn't it be enough to take out the platters and fly over them with a permanent magnet a few times (so as not to damage the platters visually, because they are nice)? —Preceding unsigned comment added by Ouro (talkcontribs) 08:18, 30 January 2008 (UTC)[reply]
Yeah, I forgot to sign, sorry. --Ouro (blah blah) 08:21, 30 January 2008 (UTC)[reply]
I think the only question this posting has demonstrated is, "How do I not destroy my hard drive?" x42bn6 Talk Mess 13:09, 30 January 2008 (UTC)[reply]
Bruce Schneier's classic book Applied Cryptography begins "There are two kinds of cryptography in this world: cryptography that will stop your kid sister from reading your files, and cryptography that will stop major governments from reading your files. This book is about the latter". But let's be honest here, Gull Man: you just need to keep out your kid sister. I'd say if you were to just poop on it and throw it in the trash, there is almost surely no chance of it ever being touched again. --Sean 14:18, 30 January 2008 (UTC)[reply]
Open the drive, remove the disks and file them down by scratching them on concrete or the road.--155.144.251.120 (talk) 22:57, 30 January 2008 (UTC)[reply]
Nuke it from orbit, it's the only way to be sure. Cheers, WilyD 23:05, 30 January 2008 (UTC)[reply]
(after edit conflict) Wow. This is a busy thread. Who knew that gratuitous violence against electrical devices was so popular these days? ;) Thanks muchly for all the tips. However I end up disposing of it (the prospect of getting to use poo for a useful purpose is an intriguing one), I think I'll open the thing up to take a look inside first. I always just figured that the contents were a thicker version of the matter you'd see inside a floppy disc. --Kurt Shaped Box (talk) 23:11, 30 January 2008 (UTC)[reply]
If you are looking for another useful purpose for poo, you may be interested in humanure. DuncanHill (talk) 23:20, 30 January 2008 (UTC)[reply]
I've always considered flinging poo to be the earliest (and most basic) debating technique. --Kurt Shaped Box (talk) 23:34, 30 January 2008 (UTC)[reply]
The best way to destroy a hard drive beyond repair is to save the only copy of your most important file on it. Second best is any of the following: smashing it with a stress-relief device, covering the disk in a pretty substance, running it through a shredder, or throwing it in an Aperture Science Emergency Intelligence Incinerator.-RunningOnBrains 10:29, 1 February 2008 (UTC)[reply]
Open it up. Plug it in. Turn it on. Wait. Repeat as necessary. If this fails touch drive with whatever you feel like. —Preceding unsigned comment added by Shniken1 (talkcontribs) 15:22, 1 February 2008 (UTC)[reply]
Put it in a plain box, store somewhere safe, and wait for entropy to run its course. SpinningSpark 22:52, 1 February 2008 (UTC)[reply]

(undent)However original you all think you have been . . . YouTube was there first [14], done it, filmed it. I particulary liked the comment "thermite is always the answer", but then again, YouTube is not trying to answer such a wide range of questions as the Science Desk. SpinningSpark 23:14, 1 February 2008 (UTC)[reply]

I've always just writen over every sector, taken the drive apart, taken out the neodymium magnets, wiped them across the platters then scored them up with a screw driver or something abrasive. I have read that with modern harddrives just over writing the data is enough to make recovery very difficult. I guess the word "difficult" depends on who you and a little mindless violence towards technology is great therapy for the frustrated modern day man. A word of warning, the smaller 2.5inch disks used in laptops that I have destroyed contained glass platters yet look metallic - if you pry them or treat them too rough they will "energetically disassemble" into small sharp pieces! —Preceding unsigned comment added by 78.32.138.240 (talk) 19:08, 2 February 2008 (UTC)[reply]

s z qasim

who is s z qasim?Zikrullah (talk) 11:35, 30 January 2008 (UTC)[reply]

My Googling tells me he is supposed to be a scholar devoted to studying ocean ecosystems, the Indian Ocean in particular. We don't have an article on him, though. --Ouro (blah blah) 12:49, 30 January 2008 (UTC)[reply]

CPR

Is is possible for a person perform CPR on them self to jump start their heart? —Preceding unsigned comment added by Hyper Girl (talkcontribs) 11:42, 30 January 2008 (UTC)[reply]

How would you go about moving your arms and applying force to your own chest while unconscious? --Ouro (blah blah) 12:45, 30 January 2008 (UTC)[reply]
Apparently, by coughing: Cpr#Self-CPR jeffjon (talk) 13:48, 30 January 2008 (UTC)[reply]
I don't see how you can even cough... Your heart can beat without you breathing (although not for long), but you can't breathe if your heart isn't pumping. Once your heart stops pumping, or goes into fibrillation, you go unconscious within a matter of seconds. -- MacAddct  1984 (talk &#149; contribs) 15:10, 30 January 2008 (UTC)[reply]
The CPR article briefly mentions the idea of self-cpr, but says it's largely hoax and myth. CPR#Self-CPR APL (talk) 13:53, 30 January 2008 (UTC)[reply]
But you can give yourself the Heimlich Maneuver, so if you're on your own it's advisable to choke on something rather than have a heart attack. --Sean 14:20, 30 January 2008 (UTC)[reply]
Surely this is a very nice paradox. If you can perform CPR on yourself you don't need it. Can I claim this as "Avery's Paradox" Richard Avery (talk) 16:21, 30 January 2008 (UTC)[reply]
It's like when someone tells you they're choking, they're not really choking. I think Mr. Liar already beat you to it, calling it the Liar paradox. -- MacAddct  1984 (talk &#149; contribs) 16:51, 30 January 2008 (UTC)[reply]
.. with a touch of Catch-22 AndrewWTaylor (talk) 18:15, 30 January 2008 (UTC)[reply]
And even April Fools Day - it seems to be one of those weird questions I might have posted on APril 1. I would never do anything to the articles, but this seems like the best place to pul a funny little joke and asking a paradoxical question like that would be the way to do it. —Preceding unsigned comment added by DTF955 (talkcontribs) 21:47, 30 January 2008 (UTC)[reply]

Destroying matter

Is antimatter the only way matter can be destroyed and release energy (aka e=mc^2)? 64.236.121.129 (talk) 16:38, 30 January 2008 (UTC)[reply]

I guess theoretically. From antimatter: "mixing of matter and antimatter would lead to the annihilation of both in the same way that mixing of antiparticles and particles does, thus giving rise to high-energy photons (gamma rays) or other particle–antiparticle pairs. The particles resulting from matter-antimatter annihilation are endowed with energy equal to the difference between the rest mass of the products of the annihilation and the rest mass of the original matter-antimatter pair, which is often quite large." -- MacAddct  1984 (talk &#149; contribs) 17:28, 30 January 2008 (UTC)[reply]
No, both nuclear fission and nuclear fusion convert mass to energy. (Also, television destroys brain matter without any energy release.) Clarityfiend (talk) 19:16, 30 January 2008 (UTC)[reply]
No (with a possible yes). Nuclear fusion, nuclear fission, and even your generic chemical reaction all exchange energy for mass. If it's exothermic, then the mass of the end product is less than what you started with, and vice versa if endothermic. All of these can be accomplished without an antimatter reagent. As for the possible yes, it may be that at some quantum level all this is accomplished by short-lived antimatter particles -- I have no understanding either way, so I don't want to rule it out -- but I don't think this is the core of what you're asking. — Lomn 19:17, 30 January 2008 (UTC)[reply]
It's also probably the only way to get complete annihilation of matter - all the other methods have the energy release as part of a bigger process that produces more matter as well. Confusing Manifestation(Say hi!) 21:41, 30 January 2008 (UTC)[reply]
Fission and fusion don't create more matter—they end up with less matter, overall, in the end. "Annihilation" doesn't really get rid of matter, it just converts it totally into (various forms of) energy. So the answer is "no", but matter-antimatter mixing is certainly the most efficient way to convert matter into energy. --24.147.69.31 (talk) 23:36, 30 January 2008 (UTC)[reply]
Once fusion reaches iron, it requires an energy input and thus produces mass. As far as I know, though, you're correct about fission. — Lomn 06:02, 31 January 2008 (UTC)[reply]
How can that be so? Aren't fusion and fission inverse processes? Wouldn't the fission of elements lighter than iron also require energy input and produce mass? --NorwegianBlue talk 15:50, 31 January 2008 (UTC)[reply]
Beats me, though that sounds reasonable. I'm only aware of fission in the context of heavy elements, but perhaps this is why. — Lomn 16:55, 31 January 2008 (UTC)[reply]
Yes, that's correct. If you hit a light nucleus with a very energetic neutron you can fission it into even lighter nuclei, but the sum of the products' masses will be greater than the mass of the original particles. TenOfAllTrades(talk) 17:15, 31 January 2008 (UTC)[reply]
More generally, each kind of nucleus has a binding energy which you can think of as the energy released if you could start with all the component protons and neutrons, and "assemble" the nucleus from the pieces. Here is a graph. Note that the general trend of the curve is increasing towards the middle where iron is. That means in general, fusion of light elements and fission of heavy elements will yield energy (and lose mass). But note that the curve is bumpy in places, so the general rule doesn't always hold. JohnAspinall (talk) 21:31, 31 January 2008 (UTC)[reply]
Potential other ways to destroy matter are the black hole, the Big Crunch, the Big Rip. Graeme Bartlett (talk) 03:17, 31 January 2008 (UTC)[reply]

Auderiense

What could the name Leptictidium auderiense refer to? I have though of the possibility of it being a reference to the Aude department in France, but it's highly improbable due to the fact that this animal has been found only in Germany. Also, if it was the Aude, it would be something more like "audiensis", wouldn't it? What could it be? Thanks. -- Leptictidium (mammal talk!) 17:24, 30 January 2008 (UTC)[reply]

While I don't wouldn't to encourage crossposting, you may actually be better off posting on the Language Reference desk. -- MacAddct  1984 (talk &#149; contribs) 17:31, 30 January 2008 (UTC)[reply]
Thanks, I will do that. I was hoping some expert in paleontology could give me an answer here. -- Leptictidium (mammal talk!) 19:51, 30 January 2008 (UTC)[reply]

How far into the ear canal is the ear drum located?

^topic 64.236.121.129 (talk) 20:55, 30 January 2008 (UTC)[reply]

The ear canal is approximately 26 mm long, with the ear drum located at the end of that, as discussed in the ledes of the articles. — Lomn 21:11, 30 January 2008 (UTC)[reply]
Actually only the ear canal article says "approximately 26 mm", thus immediately showing that the number is untrustworthy. The other one doesn't say.
The old edition of Gray's Anatomy available at bartleby.com says here that the distance along the ear canal (external acoustic meatus) from the bottom of the concha (the central cavity of the external ear) to the tympanic membrane (eardrum) is about 2.5 cm (25 mm), or from the tragus (the little flap you can press on to "close" your ears) to the eardrum, about 4 cm (40 mm).
--Anonymous, 00:18 UTC, January 31, 2008.
I fail to see how no discussion equates to "untrustworthy". The eardrum article doesn't mention earwax either (ear canal mentions it in the context of the eardrum); is its existence likewise in doubt? Nonsense. Now if one article said 26mm and the other said 1cm, there might be a trust concern -- but they don't, so there isn't, particularly when Gray's agrees to within 1 millimeter -- a number I suspect is well within the range of normal human variation. — Lomn 05:59, 31 January 2008 (UTC)[reply]
No, it's "approximately 26" that implies untrustworthiness. It's an obvious case of false precision and therefore implies that the passage was written carelessly. --Anon, 23:48 UTC, Jan. 31.
I would disagree that two significant digits are an obvious case of false precision. Perhaps the text can be more clear, but that does not throw its veracity into doubt. — Lomn 22:39, 1 February 2008 (UTC)[reply]

Pet Parrots need 12 hours or more sleep a night?

Is this true? The internet seems to think so. My macaw (hyacinth) doesn't get anywhere near that. She lives with me in a one room flat, so she's pretty much awake when I am (she's usually awake before me, I guess when she hears the birds singing outside). Will this be affecting her health? I've had her for many years and she seems okay. Thanks for the tips about macaw training by the way (the question I asked a few days ago). --84.66.26.102 (talk) 23:51, 30 January 2008 (UTC)[reply]

I would think that parrots are like humans in that they probably need so many hours of sleep (8 for us) but many people get by fine with less. I haven't had a good 8 hours sleep in absolutly ages. Could it be that she is sleeping at times other than whenshe normally sleeps as well? TheGreatZorko (talk) 08:46, 31 January 2008 (UTC)[reply]

It seems this may explain it. One part of the brain can sleep while the other is awake. TheGreatZorko (talk) 08:54, 31 January 2008 (UTC)[reply]

(after edit conflict) 12 hours seems like a lot. I don't know how accurate that is - there's a lot of birdcare advice on the internet that's not really worth much. My experience with parrots tells me that they'll sleep when they're tired. If she's alert and wanting to play/hang out, then she's not tired. --Kurt Shaped Box (talk) 09:08, 31 January 2008 (UTC)[reply]
Pet carrot = Parrot? LOL - reminds me of a personal joke where Bone marrow = Barrow. Sandman30s (talk) 11:30, 31 January 2008 (UTC)[reply]
Looks like an annon user went and changed the title. I put it back.

feel like crap

What causes the really bad feeling while being sick, such as having the flu or common cold? You know when you get out of bed to take your medicine and you go "ugh... I feel like crap!" and take your meds and go back to sleep. I hope someone know's what I'm talking about. schyler (talk) 23:54, 30 January 2008 (UTC)[reply]

Ah, malaise! - Carbon [Nyan?] 23:58, 30 January 2008 (UTC)[reply]
I have the notion that it's interferon that makes you feel like that, and that the reason it makes you feel like that is precisely so that you will go back to sleep, conserving energy for the immune response. But I'm not really sure about either part. --Trovatore (talk) 00:02, 31 January 2008 (UTC)[reply]

Yep. That's it. Thanks. schyler (talk) 00:36, 31 January 2008 (UTC)[reply]

It is prostaglandins, that's also the reason why non-steroidal anti-inflammatory drugs help against that sicky feeling. Prostaglandins are probably downstream of interferon. Сасусlе 04:30, 31 January 2008 (UTC)[reply]

January 31

Cause of gravity- what is it?

I was watching a tv program a few days ago where Dr Brian Cox (quantum physicist) first said gravity was due to the bending of spacetime as per Einsteins theory, but then said it was due to gravitons which haven't yet been seen. So which is it really--TreeSmiler (talk) 00:57, 31 January 2008 (UTC)[reply]

An experiment in the new supercollider is supposed to find proof of gravitons and strings. I thought it was supposed to already have taken place, but we would have heard about any proof or lack of proof found. Personally, I do not believe in gravitons in any way. It isn't a scientific theory based on experiment. It is just that there is nothing observed in gravity that requires gravitons. Therefore, I see no reason to assume that gravitons exist. -- kainaw 01:14, 31 January 2008 (UTC)[reply]
So you think its bent spacetime?--TreeSmiler (talk) 01:19, 31 January 2008 (UTC)[reply]
It's not so clear that the two things are different. Ideally a theory of quantum gravity would be something that describes gravity such that, when you look at it one way, it's an exchange of virtual gravitons, and when you look at it another way, it's curvature in the metric tensor. As far as I know, no one has yet come up with anything very satisfactory along those lines, but that's what I think physicists want. --Trovatore (talk) 01:24, 31 January 2008 (UTC)[reply]
This is like asking whether a zebra is black with white stripes or white with black stripes. When the electrons in an antenna wiggle back and forth in response to a passing radio wave, is this because they are responding to oscillations in the electromagnetic field or because they are interacting with a stream of photons ? It's both - these are just two different descriptions or models of the same physical phenomena. In the same way, you can describe gravitational force as a bending of spacetime or as an exchange of gravitons. For almost all purposes the bending of spacetime model is adequate and simpler to use (in fact, for many purposes, the "force at a distance" model of Newtonian gravity is adequate and simpler still). It's only when you are looking at very high energy densities and very small scales that the behaviour and interaction of individual gravitons becomes significant. This is when you need a consistent theory of quantum gravity (which we don't yet have). Gandalf61 (talk) 10:46, 31 January 2008 (UTC)[reply]
  • Switching from physics to zoology, if you look at how the stripes end you can see that zebras are white with black stripes. Sorry, couldn't resist. --Anon, 07:30 UTC, February 1, 2008.
Unfortunately for the theory, gravitons (unlike photons) have yet to be detected. Why is that?--TreeSmiler (talk) 13:50, 31 January 2008 (UTC)[reply]
Perhaps because we don't have the right instruments, or the right experimental setup, or perhaps because they don't exist. No firm answer for this can be given. You may be interested in reading about the Higgs boson, a theorized particle with much wider acceptance than the graviton which is also still undetected. — Lomn 14:24, 31 January 2008 (UTC)[reply]
Our graviton article says that gravitons (if they exist) interact so rarely with matter that it would be very difficult (and perhaps impossible) to build a detector that could directly detect individual gravitons. I imagine this is connected with the fact that gravity is very much the weakest of the four fundamental forces. The best we can do with present day technology is to try to detect gravitational waves with experiments such as LIGO. Finding out more about the properties of gravitational waves might allow us to confirm or rule out the existence of gravitons. Gandalf61 (talk) 14:22, 31 January 2008 (UTC)[reply]
Hmm. If gravitons dont interact with matter much, how can they be responsible for the effect of gravity. Yes I believe LIGO has had no success either. So if gravity waves dont exist either, what then?--TreeSmiler (talk) 16:13, 31 January 2008 (UTC)[reply]
Gravitons don't need to interact much with matter because gravity is very very very weak compared to the other fundamental forces. If gravity were as strong as the electromagnetic force then anything with a mass more than a few grammes would collapse into a tiny ball of neutronium under its own self-attraction.
We know, indirectly, that gravitational waves exist because the orbital decay curve of certain binary star systems can only be explained if the system is radiating gravitational energy in accordance with the predictions of general relativity - see binary pulsar and Hulse-Taylor binary. What we haven't done yet is detect a specific gravitational wave event. An analogy is that you know someone is raiding your fridge if food goes missing, even if you haven't caught the culprit in the act. Gandalf61 (talk) 16:48, 31 January 2008 (UTC)[reply]
So if its invisible gravitons and gravity waves, how does space time get bent by these?--TreeSmiler (talk) 02:00, 1 February 2008 (UTC)[reply]
A gravitational wave doesn't cause the bending of spacetime, it is the bending of spacetime. An analogy is that a water wave doesn't cause a disturbance in the water, it is the disturbance in the water. A water wave is a time-dependent change in the water level, and a gravitational wave is a time-dependent change in the curvature of spacetime.
Gravitons are an alternative view or model of the same thing - when a particle of matter interacts with a passing graviton, it behaves as if the curvature of spacetime in its vicinity had changed. The problem with most versions of the graviton model is that gravitons also interact with other gravitons, giving rise to runaway calculations that lead to "infinities".
I realise this is all a bit vague, and answers that describe gravitational waves and gravitons as if they were water waves and tiny ball bearings are bound to be incomplete and inaccurate at some level. To get the real McCoy you have to get into the advanced mathematics of Riemannian geometry, general relativity and quantum field theory, which is a long and difficult journey. Gandalf61 (talk) 10:15, 1 February 2008 (UTC)[reply]
Yes but we can see the so called bending of space time by the orbits of moons and planets and by gravitational lensing, but we cant see either the gravitational waves, or the gravitons that 'cause'it. Is that what you are saying? Hmmm!--TreeSmiler (talk) 17:52, 1 February 2008 (UTC)[reply]

I must point out that gravitons are entirely hypothetical. They are no more factual than tachyons, in the sense that they don't violate any scientific laws, but don't necessarily exist. Einstien's theory of general relativity covers why gravity occurs. In short, objects with mass make dents in space-time. The more massive the object, the larger the dent is. The larger the dent, the larger the effect of gravity is. 64.236.121.129 (talk) 17:09, 1 February 2008 (UTC)[reply]

Gravitons are indeed hypothetical, but they seem to a hypothesis with a high degree of necessity. We know for certain that general relativity is not the last word on the subject of gravity, because it is an entirely classical (i.e. non-quantum) theory. Some form of graviton is a key component of each of the currently proposed flavours of quantum gravity, AFAIK. Our quantum gravity article says: "If the graviton turns out not to exist, it will render all work based on quantized macroscopic physics flawed, and destroy virtually all the accepted notions of a unified theory of physics since the 1970s". Gandalf61 (talk) 17:55, 1 February 2008 (UTC)[reply]
So gravitons are to gravity, what photons are to light (em radiation). And gravity waves are to gravity, what em waves are to em radiation. BUT gravity waves are so weak and feeble, and gravitons are so rare, that we have difficulty in detecting either of them? Is that right?--TreeSmiler (talk) 18:02, 1 February 2008 (UTC)[reply]
Yes, I would say you have got it right. One minor correction - it is not the gravitons themselves that are rare, it is their interactions with matter (which is the only way we can detect them directly) that are rare. In physics-speak we say that gravitons have a very low cross section. Gandalf61 (talk) 09:11, 2 February 2008 (UTC)[reply]

Quantum Physics

I have an equation here in which I am having a very hard time getting both sides of the equation to equate to one another. The answers I have been getting are radically different where they should equate. This equation is supposed to relate the radius of a quantum dot to the frequency of light it emmits. I ordered this kit from Cenco Physics so I assume all of the values are correct. It would be great if someone could point out how to get the sides to equate. (Please include steps).

In which:

Dirac's constant

wavelength

Planck's constant

radius

speed of light in vacuum

For clarity:

Joules

nanometers

kilograms

Zrs 12 (talk) 01:41, 31 January 2008 (UTC)[reply]

It seem to comes out about right with n=1. I get about 3.3×10−19 J on each side using Google Calculator. I'm not sure how to break this down into steps; you just regularize the units and then do the arithmetic. Maybe you're missing a unit conversion factor somewhere? -- BenRG (talk) 20:09, 31 January 2008 (UTC)[reply]
Thanks. What units did you have to convert, though? I have just been plugging the values in as are (which is probably the problem). Zrs 12 (talk) 20:27, 31 January 2008 (UTC)[reply]
For one thing, if you're going to use J and kg, to be consistent you'll need to use m for the radius, not nm as listed. Also, make sure you use m/s units for c, and J s units instead of eV s for h and . MrRedact (talk) 05:52, 1 February 2008 (UTC)[reply]
Oh O.K. thanks. I bet that solves the problem I've been having. Zrs 12 (talk) 14:06, 1 February 2008 (UTC)[reply]

Songs written by drummers

Why is it that songs written by drummers tend to be worse than songs written by a band's singer or musicians, anyone else in the band in other words? I know that there's been exceptions but more often than not, it seems like the tracks I often end up skipping on albums have been filler tracks written by the drummer. —Preceding unsigned comment added by 81.77.43.42 (talk) 02:28, 31 January 2008 (UTC)[reply]

There is a stereotype, of course, that drummers are stupider or less musical than the rest of the band members. I've no idea if there's any truth to that but of course there are a lot of notable exceptions. It could be that they've never been the centerpiece of the band and so have less experience with writing songs than the other members. Who knows. It might just be that the bands you listen to have drummers who are bad songwriters. --24.147.69.31 (talk) 02:54, 31 January 2008 (UTC)[reply]
I've also heard that drummers tend to be more explosive. Meanwhile, Phil Collins seems to be an argument against 81.77's theory.
Atlant (talk) 13:02, 31 January 2008 (UTC)[reply]
As is Don Henley and others. Though they are, of course, the very notable exceptions to the general rule, I think, that drummers aren't the real songwriters behind the band. --24.147.69.31 (talk) 14:30, 31 January 2008 (UTC)[reply]
Hmmm. This isn't really a scientific question, is it? At least not in the way that "Who the hell decided that making the drums sound like they were made out of rubber and wet cardboard on songs produced in the 80s was somehow cool?" is... ;) —Preceding unsigned comment added by Kurt Shaped Box (talkcontribs) 01:40, 1 February 2008 (UTC)[reply]
Our article on The Yardbirds says that their drummer, Jim McCarty, was involved with the song writing.
Atlant (talk) 13:16, 2 February 2008 (UTC)[reply]

Shower mold

What are the common species of shower mold? I am most interested in the "orange, very hard to scrub off but not rust" variety. Also can one identify molds at home? -Ravedave (talk) 02:30, 31 January 2008 (UTC)[reply]

In the U.S., both Home Depot and Lowe's sell mold detection packages that you mail in and a lab sends back a mold identification. You can do specific samples or a basic air sample. -- kainaw 02:33, 31 January 2008 (UTC)[reply]
Really? Wow. I might have to grab some of them to test my friends. SGGH speak! 11:22, 31 January 2008 (UTC)[reply]
All of our home supply stores have them in the air filter section. It is a little pack with a petri dish, instructions, and an envelope to mail it to the lab. -- kainaw 15:23, 31 January 2008 (UTC)[reply]
Your friends are molds? Clarityfiend (talk) 20:58, 31 January 2008 (UTC)[reply]
Maybe he's just not sure if his guy friends are fun or not...Tuckerekcut (talk) 22:19, 1 February 2008 (UTC)[reply]

minerals and rocks

A____________is a mixture of solid particales called grains. —Preceding unsigned comment added by 24.5.216.130 (talk) 04:42, 31 January 2008 (UTC)[reply]

Rule #2 above: "The reference desk will not give you answers for your homework", and this looks like a homework question to me. -- HiEv 04:59, 31 January 2008 (UTC)[reply]
I think we can bend the rules here, the answer is granola Mad031683 (talk) 20:29, 31 January 2008 (UTC)[reply]
I assume the "grain" here is part of the etyomogy of the word "granola"? That doesn't make any sense though, because then "villain"→"vanola", yet vanilla-white is the color of the good guy, not the villain. I knew there's a reason I went into science not language-arts. DMacks (talk) 20:36, 31 January 2008 (UTC)[reply]
bit too keen to say 'homework' above I think - could be a crossword clue.. How about "granular agglomerate".
I the original questioner could give us a little more context to the question a better answer might develope. The answer could be 'sandstone' for example?87.102.33.230 (talk) 20:44, 31 January 2008 (UTC)[reply]
Context would indeed help. Clastic rock could be the answer tho'. DuncanHill (talk) 01:29, 1 February 2008 (UTC)[reply]
A granular mixture of particles is a mixture of solid particles called grains. − Twas Now ( talkcontribse-mail ) 02:26, 2 February 2008 (UTC)[reply]

Sick again

In instances where I've gotten a cold from a friend or vice versa, I've found that increased exposure to said friend can result in a sort of relapse of sickness when I'm starting to get better. This has happened more than a few times, despite people telling me that this is not very likely and that I probably got sick from something or someone else. What's the scoop? What's the likelihood of this sort of thing happening? — Ƶ§œš¹ [aɪm ˈfɻɛ̃ⁿdˡi] 05:20, 31 January 2008 (UTC)[reply]

This is very close to the "Don't ask or give medical advice rule" but you don't seem to asking anything specific. What I'd wager is that you are giving your cold to your friend, and due to the virus' ability to mutate quickly it is mutating within his system to the point where your immune system is unable to defend against it well, causing you to catch the cold off him again. TheGreatZorko (talk) 08:42, 31 January 2008 (UTC)[reply]
It doesn't need to involve mutation. There are lots of different viruses that cause colds, and you can be getting over one of them when you catch another from your friend. - Nunh-huh 18:43, 1 February 2008 (UTC)[reply]

astrologly

What would make a star change colors rapidly?--207.32.21.29 (talk) 14:01, 31 January 2008 (UTC)Tyler J Long[reply]

I think you mean "astronomy" rather than "astrology" in the title. Viewed from Earth, a twinkling star (which can appear to change color) is exhibiting scintillation. A star like the sun can, over a long period, exhibit color changes as a function of surface temperature as outlined in stellar evolution. Finally, a nova or supernova can result in a sudden massive change in a star's appearance, color included. If, on the other hand, you really did mean "astrology", changes can be explained by anything you choose. — Lomn 14:21, 31 January 2008 (UTC)[reply]
(ec) Usually the apparent colour of a star depends on its surface temperature (the spectrum of a star is very close to that of an ideal blackbody). Our article on stellar classification delves into this in some detail.
Changes in apparent colour come about in a number of different ways. Our article on variable stars is extremely thorough. Depending on what you mean by 'rapidly', there are processes that act on timescales anywhere from minutes to years. (All might be considered 'rapid' on astronomical timescales.) TenOfAllTrades(talk) 14:27, 31 January 2008 (UTC)[reply]
Though they are not stars, some of our neighbor planets appear as stars from Earth. One of the main differences between them and stars when viewed with the naked eye is that the planets twinkle more and sometimes appear to change color. -- kainaw 15:29, 31 January 2008 (UTC)[reply]
Um, planets don't twinkle anywhere near as much as stars do. That's a good way to tell they're planets. I can't find a reference for that in Wikipedia, but if it's a clear night where you are you can see for yourself. --Milkbreath (talk) 16:35, 31 January 2008 (UTC)[reply]
Phil Plait to the rescue—here's a reference on planetary twinkling: [15]. TenOfAllTrades(talk) 17:18, 31 January 2008 (UTC)[reply]
I was typing one thing and thinking another completely. My point was that the planets appear as steady stars - no color change. This would make the stars that twinkle seem odd in comparison. But, since the planets aren't stars, the twinkling stars are normal. But, I typed the exact opposite. In my defense, I'm at home today taking care of a 13-month old with molars coming in. It is difficult to think when a baby is screaming non-stop all day. -- kainaw 18:39, 31 January 2008 (UTC)[reply]
I figured it must be something like that—pack of rabid wolves trying to get in, killer bee attack, arterial bleeding, squalling baby.... Just wanted to keep things straight for the questioner. --Milkbreath (talk) 19:03, 31 January 2008 (UTC)[reply]

virus

carried by cats that changes mouse brain —Preceding unsigned comment added by 66.32.211.150 (talk) 20:45, 31 January 2008 (UTC)[reply]

Toxoplasma gondii (not a virus). Сасусlе 21:50, 31 January 2008 (UTC)[reply]

Square drums

Why are drums round, rather than square? --Carnildo (talk) 21:19, 31 January 2008 (UTC)[reply]

I would imagine it is so that they give the same sound at equal distances from the central point, and so that the skin isn't stretched too tight at corners and things. A square has four corners which are points of weakness. It could also be for acoustic reasons. -mattbuck 21:28, 31 January 2008 (UTC)[reply]
Also, I'd say from the constructional point of view - a round drum will be stronger than a cubical drum made of similar materials. I think. --Ouro (blah blah) 21:40, 31 January 2008 (UTC)[reply]
Sometimes, they are hexagonal.  ;-) --LarryMac | Talk 21:50, 31 January 2008 (UTC)[reply]
...or octagonal. --169.230.94.28 (talk) 22:59, 31 January 2008 (UTC)[reply]
Mathematics has gone a long way. Try to read hearing the shape of a drum article. Cheers, --Dr Dima (talk) 08:27, 1 February 2008 (UTC)[reply]
There's also the traditional element - drums used to be made by stretching skin over pots or wooden bowls, which are much easier to make round (using a pottery wheel or lathe) than square. Laïka 09:44, 1 February 2008 (UTC)[reply]
One argument *for* using square drums would be that it would enable the drummer to more easily count to four without error. All you'd need then is some method of stopping him soloing inappropriately and a bucket to catch the drool.
Ahhh. What would a drum-related thread be like without at least one drummer skit? ;) --Kurt Shaped Box (talk) 13:29, 1 February 2008 (UTC)[reply]

carbon dioxide

Why cant we just put up a bunch of carbondioxide filter things and use the carbon they filter to make carbon fiber then put that carbon fiber on cars making them lighter and more fuel effitiont or houses or what ever? —Preceding unsigned comment added by 76.235.204.179 (talk) 21:54, 31 January 2008 (UTC)[reply]

In a word, expense. Filtering carbon dioxide can be costly, and manufacturing carbon fiber is certainly costly. Even with increasing eco-friendly interests, the market won't support it. — Lomn 22:41, 31 January 2008 (UTC)[reply]
As you said, they would filter out carbon dioxide, which would require a lot of effort and expense to convert into carbon fiber. They are different compounds, not just easily interconvertable forms of the element "carbon". DMacks (talk) 22:48, 31 January 2008 (UTC)[reply]
And in particular, converting carbon dioxide to carbon would consume more energy than was released by burning the oil, coal, gasoline, or whatever generated the carbon dioxide in the first place. --169.230.94.28 (talk) 22:54, 31 January 2008 (UTC)[reply]
I've done a lot of contract work for a paper company that plants tons of carbon dioxide filters (trees) and turns them into paper (which is technically carbon-based and fibrous). Of course, one of the major byproducts of the process is production of carbon dioxide. -- kainaw 01:33, 1 February 2008 (UTC)[reply]

August eclipse times

Hi. I'm just wondering, when does partiality end for the following cities for the solar eclipse on August 1, 2008, local daylight savings time?

  • Rimouski, Quebec
  • Gaspé, Quebec
  • Fredricton, New Brunswick
  • Saint John, New Brunswick
  • Halifax, Nova Scotia
  • Yarmouth, Nova Scotia
  • Sydney, Nova Scotia
  • Charlottetown, Prince Edward Island

I don't want the times for totality, because totality will not even be occuring in those reigons. I think I might be able to find this, so so far my searches with Yoursky have made the sun and moon too big, but perhaps I can find it eventually. I realize that the eclipse ends at sunrise for some of these places, so don't give me a time without mentioning that it ends before sunrise if that is the case, but I wish to know when the edge of the moon completely leaves the sun in these places. Also, if possible, try to include the elevation (altitude) of the sun at these times, but this is not nessecary because if you can give me a time, I could look up the coordinates and input the time on Yoursky. Thanks. ~AH1(TCU) 23:15, 31 January 2008 (UTC)[reply]

Fred Espenak at NASA maintains a great set of pages about eclipses. This PDF file contains all sorts of details about the eclipse. On PDF page 42 (page 34 as marked on the page), Table 9 contains contact times for various points in Canada, including five of the eight places on your list. In all five places the eclipse begins before sunrise and reaches maximum extent before sunrise, but does not end before sunrise. The specific time you're asking about is called "fourth contact". The table gives it only given in UT; I'll convert them to local time myself, but you should check the original document to make sure I haven't miscopied from it. I'll add Chicoutimi, the nearest place they give to Rimouski, which you asked about. I assume it's on Eastern Time, unlike all the other places listed here.
     Chicoutimi ....... 09:34:25.0 UTC (05:34:25.0 EDT) altitude 2°
     Charlottetown .... 09:26:01.3 UTC (06:26:01.3 ADT) altitude 4°
     Fredericton ...... 09:27:28.5 UTC (06:27:28.5 ADT) altitude 2°
     Halifax .......... 09:22:59.2 UTC (06:22:59.2 ADT) altitude 3°
     Saint John ....... 09:25:45.0 UTC (06:25:45.0 ADT) altitude 2°
     Sydney ........... 09:24:17.3 UTC (06:24:17.3 ADT) altitude 6°
Now look at Figure 2 on PDF page 55 (page 47 as marked). You might want to enlarge it to see the relevant region more easily. Clearly, the further northeast you are in this region, the more of the eclipse you see (weather permitting) time-wise, and the further northwest you are, the greater the fraction of the sun that is eclipsed. So if the weather is clear everywhere, Gaspe would be the best choice of the places you mentioned.
--Anonymous, 00:28 UTC, February 1, 2008.

February 1

pubmed error

Why can't I find this title in pubmed? "GSTP1 CpG island hypermethylation as a molecular biomarker for prostate cancer" ----Seans Potato Business 01:01, 1 February 2008 (UTC)[reply]

I found it. [16] Someguy1221 (talk) 01:16, 1 February 2008 (UTC)[reply]
What search string did you use? Why doesn't the title work? I tried the title, the title without the first few words, the main author and date... ----Seans Potato Business 07:52, 1 February 2008 (UTC)[reply]
I just copy-pasted the title into google scholar and voila, the pubmet hit was number three. Someguy1221 (talk) 10:11, 1 February 2008 (UTC)[reply]
Okay, I'll remember that for next time (but I still think that pubmed have a problem (not the first time that I've encountered it, either; I've contacted them before and was fobbed off with some rubbish). ----Seans Potato Business 12:14, 1 February 2008 (UTC)[reply]

impulse of photon

I want to know about impulse of photon on reflection from a surface.please provide with a formula. —Preceding unsigned comment added by 59.95.77.49 (talk) 06:25, 1 February 2008 (UTC)[reply]

Well I not 100% sure but...firstly you have to keep in mind that when you're dealing with photons it's not quite the same as in regular newtonian physics...after all a photon has no rest mass and doesn't 'reflect' off a surface. That aside, and assuming you mean it 'reflects' back in the opposite direction from which it came, and knowing that impulse is the change in momentum, and that the momentum of a photon is h/λ, I guess the impulse would be I = 2h/λ, where h is plancks constant and λ is the wavelength. I'm sure someone will kindly correct me if I'm wrong... Trimethylxanthine (talk) 06:47, 1 February 2008 (UTC)[reply]
Quite right. While quantum wave-particles don't interact according to Newton's laws, the various conservation rules still apply. So the important facts of simple reflection is that a photon went in with h/λ momentum, and left with -h/λ momentum. Therefore, the remainder of the system experienced a change in momentum of 2h/λ, even without knowing what really took place. Someguy1221 (talk) 10:14, 1 February 2008 (UTC)[reply]
Don't forget the angle - making it 2h sin(a)/λ where a is the angle to the reflective plane.. (is that still right?)87.102.12.64 (talk) 15:00, 1 February 2008 (UTC)[reply]
Yes; I just prefer to operate in as few dimensions as necessary ;-) Someguy1221 (talk) 15:50, 1 February 2008 (UTC)[reply]

Burning Sensation in the chest when exerting oneself

Medical question removed. It may very well be that your condition is simply due to over-exertion, but we goofs on the refdesk are not able or allowed to make that call. See your doctor. If you really think that will be a problem, see if you can talk to a nurse or pharmacist or someone at a drop-in clinic. Semi-anonymous encyclopedia geeks are not a good source of medical help! Matt Deres (talk) 19:32, 1 February 2008 (UTC)[reply]

chloroplasts

With present technologies is it possible to keep a chloroplast working (making photosynthesis) outside of a cell, in vitro? sorry if my english is not correct thank you 87.2.65.209 (talk) 17:33, 1 February 2008 (UTC)[reply]

As far as I know, yes. Since photosynthesis depends on the transport of electrons through ATP producing carrier protiens (and the electron transport chain which is controlled by light), chloroplasts can be placed in an acidic solution so diffusion will transport H ions into the chloroplast. Then, when the chloroplast is placed into a basic solution, a concentration gradient will be created between the basic solution and the high concentration of H ions in the cell. This will cause the electrons to diffuse through the ATP producing carrier proteins, thus producing ATP. Zrs 12 (talk) 20:01, 1 February 2008 (UTC)[reply]
But can the chloroplast survive outside the cell to begin with? − Twas Now ( talkcontribse-mail ) 01:21, 2 February 2008 (UTC)[reply]
I'm not sure. However, I suppose they could. See [this link] (answer number 3) and make what you will of it. It seems to me though, this link means they could and just don't. Zrs 12 (talk) 01:49, 2 February 2008 (UTC)[reply]
Yes. The endosymbiotic theory. But can we remove them and keep them alive indefinitely? It would be extremely difficult, I think, to match the environment of a cell in vitro. − Twas Now ( talkcontribse-mail ) 02:21, 2 February 2008 (UTC)[reply]
I just found another link which seems to claim it wouldn't be possible.[1] In which it says about half way down the page, "Mitochondria and chloroplasts have lost so many genes that they can't live separate from their "host cell"; but they are essentially degenerate procaryotes." However, being in vitro in an ideal environment would provide them with nutrients and other things which the cell would normally provide. So, apparently, depending on which genes have been lost, they may or may not be able to be kept "alive" in vitro. Zrs 12 (talk) 02:43, 2 February 2008 (UTC)[reply]

Chloroplasts cannot live indefinitely, which is why they are called semi autonomous organelles. They can live long enough to be useful. Some sea slugs harvest them from algae and store them in specialised pouches for their own benefit. This is known by the term Kleptoplasty. David D. (Talk) 03:07, 2 February 2008 (UTC)[reply]

Diet and prescribed drugs

Questions calling for medical advice are not allowed on the Reference Desk. Please consult your doctor. --Anon, 01:58 UTC, 2008-02-02.

Some prehistory questions

1) When the Paleo-Indians crossed the Bering land bridge, did they spend generations on the journey or did they cross the bridge and the Canadian glaciers all at once? Is this even knowable?

This could be found out (or may already have been) by radio-carbon dating the skeletons found along the supposed route of travel or by anylyzing the stone tools in order to date them (by comparision to stone tools of know age and orgin). So I would say it is probably "knowable". However, there are some current theories that say the Paleo-Indians came from Europe to the North American continent via canoe along the edge of the massive ice-cap that would have been in the Atlantic Ocean. There is also another that says the first North American inhabitants came via canoe from Russia along the edge of the ice-cap in the Pacific Ocean. Zrs 12 (talk) 01:04, 2 February 2008 (UTC)[reply]
This would have taken generations. These people had no intent to move south; they were probably following migrating game herds. Only when population stress occurred (on either the humans or on the herds) would there be a need to migrate further. If one group could cross the land bridge, then it is probable that other groups crossed after them as well. These new migrants would either have to force the current residents to move onward, or be forced to move on themselves. − Twas Now ( talkcontribse-mail ) 02:04, 2 February 2008 (UTC)[reply]
For what it's worth, I recall reading that genetic research done on native americans shows three distinct waves that people came from Asia to the Americas in and that these populations were fairly small. — Ƶ§œš¹ [aɪm ˈfɻɛ̃ⁿdˡi] 11:17, 2 February 2008 (UTC)[reply]

2) Is it really true that Africa was nearly untouched by the Pleistocene extinctions?

One theory suggests that because humanity arose in Africa, and as fledgling early Hominina were only learning to hunt, the animals of Africa adapted to avoid these primates. When early Homo left Africa, they had acquired a certain degree of hunting skills, and the fauna of other lands did not think to avoid these small little creatures. Why would a 7 tonne mammoth be scared of a 75 kg human? − Twas Now ( talkcontribse-mail ) 02:04, 2 February 2008 (UTC)[reply]

3) Are there any theories as to why there are no land mammal predators on the size scale of Tyrannosaurus or Giganotosaurus, but there are predatory whales at least as big as the Mesozoic sea predators?

Vultur (talk) 23:49, 1 February 2008 (UTC)[reply]

Well, this isn't specific to predators, but Bergmann's Rule is one explanation. The deep ocean is pretty cold, so larger animals have an advantage in the heat retention department. --Bmk (talk) 15:07, 2 February 2008 (UTC)[reply]

Winter

About how many days long is winter? —Preceding unsigned comment added by 64.119.61.7 (talk) 00:44, 2 February 2008 (UTC)[reply]

The winter of 2007-2008 is 90 days long. However, I don't know if this varies from year to year or not. Zrs 12 (talk) 00:58, 2 February 2008 (UTC)[reply]
Depends on what you mean by "winter". See Seasons#Reckoning, and that list isn't exhaustive either. --Anonymous, 01:56 UTC, 2008-02-02.

Neurontin and Lyrica

What is the difference between Neurontin (Gabapentin) and Lyrica (Pregabalin) when used to treat neurogenic/neuropathic pain? ៛ Bielle (talk) 01:04, 2 February 2008 (UTC)[reply]

Try searching wikipedia! Perhaps reading the encyclopaedia pages linked above and here, Neurontin, Lyrica, will help. The first paragraphs of each article seem to answer your query at a basic level. --Bmk (talk) 01:42, 2 February 2008 (UTC)[reply]
See Neuropathy#Treatment of neuropathic pain. hydnjo talk 01:45, 2 February 2008 (UTC)[reply]
Thank you, hydnjo. I hadn't seen that article before. Bmk's insertion of the links into my question, along with the repeat of same in his/her answer, all neatly skewered on that exclamation point, tell me I should have written out the long form of my question, which was, as follows:
Having read the articles Neurontin and Lyrica, I understand that the active mechanisms by which either works in the treatment of neurogenic/neuropathic pain are currently unknown. Is there anyone out there in Science Ref Desk Land who knows of any more recent data that would explain the differences between the drugs when used to treat neurogenic/neuropathic pain? Thank you for your patience. ៛ Bielle (talk) 04:01, 2 February 2008 (UTC)[reply]
Sorry - didn't mean to sound patronizing. I would have responded differently had I noticed that you were not an anonymous user! --Bmk (talk) 15:01, 2 February 2008 (UTC)[reply]
I am sure Bmk can't have meant that it is acceptable to be patronizing to unregistered users just because they are unregistered. ៛ Bielle (talk) 16:05, 2 February 2008 (UTC)[reply]
Naturally not - I merely meant that such a comment would be less likely to be taken as patronizing for an unregistered user, as they are less likely to be aware of the rich informational content of wikipedia. --Bmk (talk) 17:23, 2 February 2008 (UTC)[reply]
I thought I could hear the crackle of ice under the surface pressure of a fast-moving body, but it turned out to be the faint sounds of far-off laughter. Excellent recovery, Bmk; Toller Cranston would be proud of you. ៛ Bielle (talk) 18:02, 2 February 2008 (UTC)[reply]

Speed of gravity

Has anyone conclusively determined the speed at which the gravitational effect travels?--TreeSmiler (talk) 01:53, 2 February 2008 (UTC)[reply]

I assume, if gravity is the effect due to the curving of space by mass (as theorized by Einstein), then gravity would not "travel". It would take effect from any distance instantly. If gravity is indeed carried by "gravitons" as quantum mechanics has hypothesized, then the effect would probably travel at the speed of light, as gravity has an infinite range and therefore its paticles must be massless. However, I am not sure of this and am just trying to put together things I have heard; so if I am wrong, someone please correct me. Zrs 12 (talk) 02:10, 2 February 2008 (UTC)[reply]
See Speed of gravity. hydnjo talk 02:15, 2 February 2008 (UTC)[reply]
That is, at exactly the speed of light. This has been experimentally verified. 70.162.25.53 (talk) 02:52, 2 February 2008 (UTC)[reply]
So would that mean Einstein was wrong about the curving of space by mass? If not, how would curvature "travel" at any speed? Zrs 12 (talk) 03:00, 2 February 2008 (UTC)[reply]
Negative. It'd mean Einstein was right - speed of gravity would be an issue of how quickly changes in mass positions propagate. Indeed, Special Relativity demands that in general, information can only travel at most at the speed of light - instantaneous gravity would cause all sorts of simultaneity-related contradictions.--Fangz (talk) 03:18, 2 February 2008 (UTC)[reply]
Indeed. As our speed of gravity article says: "General relativity predicts that gravitational radiation should exist and propagate as a wave at the speed of light". And this is entirely consistent with the existence of massless gravitons, which are hypothesised by the various competing theories of quantum gravity. Gandalf61 (talk) 09:23, 2 February 2008 (UTC)[reply]
I see. The speed of gravity in the theory of general relavtivity is the speed at which the new curve of space due to a change in mass travels away from the body. Right? Zrs 12 (talk) 15:27, 2 February 2008 (UTC)[reply]
Yes. The speed of gravity is the speed at which changes in the gravitational field propogate, and the gravitational field determines the curvature of spacetime. Gandalf61 (talk) 17:07, 2 February 2008 (UTC)[reply]

February 2

Sky

Figure showing the more intense scattering of blue light by the atmosphere relative to red light.

Why is the sky blue? 143.43.29.48 (talk) 02:25, 2 February 2008 (UTC)[reply]

Rayleigh scattering of sunlight leading to diffuse sky radiation. Rockpocket 02:33, 2 February 2008 (UTC)[reply]
I've also gotten that far, but why does air scatter blue light more than the other wavelengths? --Sean 17:50, 2 February 2008 (UTC)[reply]
Hmm, maybe the oxygen atoms in the air scatter more blue than all the other colours scattered elsewhere during a clear day? Thanks. ~AH1(TCU) 18:26, 2 February 2008 (UTC)[reply]
When Rayleigh scattering of light occurs, the particles doing the scattering (in the air) are smaller than the wavelength of the light being scattered. Because of this, the wavelength of the light becomes a huge factor in determining how much scattering occurs. (Think about it like this: light with a large wavelength has a better chance of "dodging" the small particle than light with a smaller wavelength. Specifically the scattering varies as an inverse of the fourth power of the wavelength. Therefore the smaller the wavelength, the more is scattered.) As the figure to the right shows, blue light is at the short wavelength end of the visible spectrum, therefore it is more strongly scattered in the atmosphere than long wavelength red light. This is why the sky typically looks blue. Rockpocket 22:49, 2 February 2008 (UTC)[reply]

Getting back into a normal sleep pattern, ideas?

Im not sure about your various countries but here in my country we get about 7 weeks off from school over christmas and into janurary. Almost every day ive been going to bed at 4am and waking up at 2pm. Now today is saturday and i start monday and i ususally wake up at 7:30ish any ideas to help me snap back into my normal sleep pattern. Someone suggested that i dont sleep tonight, so it will be easier for me to sleep tomorrow, wonder if this will work..hmm. Anyway any ideas? =] . —Preceding unsigned comment added by 121.219.227.98 (talk) 06:46, 2 February 2008 (UTC)[reply]

That’s the kind of metaturnal sleep schedule I too engage in when given the opportunity. :) If you had a week to do it there is a less painful way, but if you have to be at school on Monday staying awake all night may be your only option. (Don’t plan anything big for tomorrow.) Alternatively you could just go to school with only three hours of sleep. --S.dedalus (talk) 07:29, 2 February 2008 (UTC)[reply]

Yeah it does but you will sleep longer than normal. Plan for 12+ hours sleep on Sunday night —Preceding unsigned comment added by Shniken1 (talkcontribs) 13:05, 2 February 2008 (UTC)[reply]

Hi. Not sleeping for a whole night is a pretty bad idea, which is why I've never done it, ever. Sleep is nessecary because the body needs rest, and it also helps regulate growth, which is why teenagers and children need more sleep than adults. Fluctuating your sleep patterns in a short period of time is not very good either, and in extreme cases you may suffer from sleep irreglarities similar to that of jet lag. However, I've read that Inuits often get six hours of sleep in summer, and fourteen in winter, but that's over a long period of time and I assume they're not on a very tight schedule. I've also heard of people who sleep and wake 40 minutes later each day than the previous one, in order to keep up with the Martian sol so they cn track the rovers. Whatever you do, please use common sense as getting a lack of sleep on your first day of school can cause you to become very tired and lose concentration, especially if you have a test, but usually they don't give you a big test on the first day of school after a long time of holiday. Anyway, I guess you're pretty lucky to have 7 weeks of holidays in winter, where I live the December-January and summer vacation combined only total about 12 weeks. Hope this helps. Thanks. ~AH1(TCU) 18:40, 2 February 2008 (UTC)[reply]

Lava lake in Krakatau?

Google maps image of the volcano with an orange/red spot in the caldera. (Not unlike this image of Mt. Erebus.) I've seen news about its recent activity but nothing mentioned a persistent lake. Anynobody 09:37, 2 February 2008 (UTC)[reply]

Hi. I'm not sure about the lake in particular, but I'm noticing something very unusual about the image. Between the orange spot and the southern part of the island that appears to be shrouded in smoke, is a bluish smoky reigon on the west flank of the volcano. If you look closely you can read the bluish-grey numbers "2008" on that area. Were these images taken this year? Is this true for the other reigons as well? Have they updateded Google Earth with new images? Thanks. ~AH1(TCU) 18:46, 2 February 2008 (UTC)[reply]
It looks like 2006 to me. Saudade7 20:46, 2 February 2008 (UTC)[reply]
Anyway, the year on a copyright notice is not necessarily the year the photo was taken. They could be claiming copyright on the specific version of it that you're seeing (whether that's valid or not, which would be a legal question and not appropriate here). It's certainly true that Google Maps/Earth images may be a few years old; I've seen that by looking at images of places near where I live, where there have been changes I can look for. --Anonymous, 23:22 UTC, 2008-02-02.

Antimatter

I've heard about antimatter but I don't know what it is (please use short words!). 86.0.85.244 (talk) 09:45, 2 February 2008 (UTC)[reply]

It is the same thing as matter, except that the electrons have positive charge, and the protons have negative charge. Positively charged electrons are called positrons, and negatively charged protons are called antiprotons. It is also my understanding that neutrons are replaced by antineutrons, see Antimatter#Antiuniverse. Check out the antimatter article for more details. --NorwegianBlue talk 10:49, 2 February 2008 (UTC)[reply]
That's correct as far as it goes, but we can add another level of detail. The difference between a particle and its antiparticle is not just a change of sign in their charge, otherwise every neutral particle, such as a neutron, would be its own antiparticle. Leptons such as the electron and the neutrino have a lepton number of 1; their antiparticles have a lepton number of -1. Baryons such as the proton and the neutron have a baryon number of 1; their antiparticles have a baryon number of -1 (in fact, the situation with baryons is a little more complicated, as they actually consist of fundamental particles called quarks, which have antiparticles called antiquarks). Lepton number and baryon number are two types of quantum numbers. Particles which have a lepton number of 0 and a baryon number of 0 are their own antiparticles - these include photons and particles such as the neutral pion which are a bound state of a quark and its antiquark. Gandalf61 (talk) 12:07, 2 February 2008 (UTC)[reply]
We have an article about antimatter at the Simple English Wikipedia. (EhJJ) 12:20, 2 February 2008 (UTC)[reply]

Insect in a vacutainer?

What would happen were i to put an instect or a spider, say, in a vacutainer with the different chemicals in side them? E.g. clot accelerator, lithium heparin, Potassium EDTA, sodium citrate, etc? Otehr than maybe die, but would it like.. dissolve, etc?

Regards. —Preceding unsigned comment added by 79.67.251.46 (talk) 14:57, 2 February 2008 (UTC)[reply]

You mean letting the air enter the vacutainer first, then putting the arthropod inside? Yes, it would die, eventually. No, it won't dissolve. Chitin-protein exoskeleton of an arthropod is quite hard to dissolve. Why do you expect anticoagulants (heparin or citrate) to dissolve an arthropod?! If the cuticula is broken in any place, some haemolymph would leak out (maybe more than usually because of the anticoagulant), but that's it. So no, the arthropod won't dissolve. Actually, I'm not even sure heparin has any effect at all in arthropods, or any invertebrates for that matter. Haemolymph is quite different from mammalian blood. --Dr Dima (talk) 15:31, 2 February 2008 (UTC)[reply]

monoclonal antibodies

why are only mice used in production of monoclonal antibodies? or are any other animals being used for the same and why?

Have you read Monoclonal antibody ? --Dr Dima (talk) 15:37, 2 February 2008 (UTC)[reply]

smell of eggs and farts

what is it that gives certain types of farts and eggs their pleasant odour? a certain chemical maybe?--Sonjaaa (talk) 15:56, 2 February 2008 (UTC)[reply]

Hydrogen sulfide.--The Fat Man Who Never Came Back (talk) 16:00, 2 February 2008 (UTC)[reply]

Plants

If the leaves of a plant didn't lose water through diffusion, the water taken in by its roots wouldn't be able to travel up the plant because of the low gradient. Would it die? xxx User:Hyper Girl 16:05, 2 February 2008 (UTC)[reply]

Plant leaves are never completely watertight, but a cactus, for example, loses very little water. Like a cactus, your plant would simply stop taking up water once it held as much as it could. This is an example of homeostasis. The plant wouldn't die because it still has water inside it, and could always draw some up more from the roots when needed. Think outside the box 16:17, 2 February 2008 (UTC)[reply]
The water does not only travel up the stem of the plant, it also travels down. So no, the movement of fluid in the plant would not necessarily cease. Not all plants are capable to survive even the temporary loss of leaves; but the defoliated plant dies not (or not only) because of the slow-down of nutrient transport from the root. So let's assume the leaves are still there. I do not know if it is possible to block water emission through leaves but not to block photosynthesis and respiration. Perhaps, simply raising humidity to 100% would work for some plants; others will "bleed" excess water through the leaf pores. It mostly depends on the ambient temperature if 100% humidity would kill a plant or not: if it's hot, death will occur quite fast due to overheating. If it's not too hot, death may, indeed, occur (much slower) due to excessive water intake and/or chemical imbalance. To summarize, I think the answer is "it depends on the choice of plant, the choice of environment conditions, and the choice of the way to block water emission". Hope this helps. --Dr Dima (talk) 17:16, 2 February 2008 (UTC)[reply]

What is Gravity?

I have asked around, and no one can give me a straight answer. Some people say it's a force, some some say it's an acceleration and I'm completely confused. Help! KarateKid101 (talk) 16:36, 2 February 2008 (UTC)[reply]

it's a force that causes things to accelerate.87.102.44.109 (talk) 17:07, 2 February 2008 (UTC)[reply]
A common confusion is defining what is meant by the word "gravity." Are you referring to the "effect of gravity" or the "cause of gravity?" The effect of gravity is a force that causes mass to be naturally attracted to other particles of mass. The cause of gravity is unknown. -- kainaw 17:38, 2 February 2008 (UTC)[reply]
You'll never get a straight answer to this question for the same reason you'll never get a straight answer to "what is multiplication?" or "what is a chair?". The only way to understand what something is is to work with it for a while until you get an intuition for its behavior. We can tell you things about gravity, but there are a lot of things one can say about gravity and everyone will tell you something different. There's probably an answer that would enlighten you, but without knowing the nature of your current understanding it's very hard to guess what that answer might be. — BenRG (talk) 19:21, 2 February 2008 (UTC)[reply]
Here's a quote from our Forces article: "The definition of force is sometimes regarded as problematic, since it must either ultimately be referred to our intuitive understanding of our direct perceptions, or be defined implicitly through a set of self-consistent mathematical formulae." I guess one could argue, based on general relativity, that gravity is special and might present conceptual problems that the other forces don't... but I get the impression, without knowing enough about physics or philosophy, that most philosophers of science feel that all forces present similar challenges to our ideas about explanation. But the original questioner seemed to be confused about force vs. acceleration, so hopefully 87.102.44.109 cleared that up. Sometimes people refer to "g" (as in 9.8m/s^2) as "gravity", but that's simply wrong. --Allen (talk) 20:10, 2 February 2008 (UTC)[reply]
It's a force. However, gravity can be replicated by acceleration. For example, if one were standing in something accelerating at 1g (9.8 m/s^2), that would be equivelent to standing on earth. Gravity is also a cause of acceleration. As far as the cause of gravity: It is described as virtual particles (gravitons) by quantum physics and as the curving of spacetime by matter in the relativity theory. Zrs 12 (talk) 00:26, 3 February 2008 (UTC)[reply]

Diet question

This is an academic question and does not seek medical advice. Does eating a diet high in saturated fats completely or only partially negate the positive effects of statins? —Preceding unsigned comment added by TreeSmiler (talkcontribs) 17:16, 2 February 2008 (UTC)[reply]

I assume you understand that statins work by blocking the cholesterol-producing enzyme in the liver. Depending on the dose, statins can block up to 60% of cholesterol production. If a person did not change his or her diet, total LDL cholesterol would still decrease because the liver would produce less cholesterol. Statins also change the ration of LDL to HDL. A higher percentage of HDL is beneficial. All in all, reducing cholesterol intake will provide a benefit of lowered total cholesterol. Statins will reduce LDL further and increase the HDL ratio. -- kainaw 17:36, 2 February 2008 (UTC)[reply]
no I didnt know that. And thank you. That all I wanted to know.--TreeSmiler (talk) 17:39, 2 February 2008 (UTC)[reply]

From the article of CFLs: "Integrated lamps combine a tube, an electronic ballast and either a screw or bayonet fitting in a single CFL unit. These lamps allow consumers to easily replace Incandescent lamps with CFLs." From the article on electronic ballasts: "Electronic ballasts usually change the frequency of the power from the standard mains (e.g., 60 Hz in U.S.) frequency to 20,000 Hz or higher, substantially eliminating the stroboscopic effect of flicker (100 or 120 Hz, twice the line frequency) associated with fluorescent lighting (see photosensitive epilepsy)."

Is in then a safe bet to assume that CFLs simply do not flicker? Or, rather, flicker at an insanely high rate that simply cannot cause problems (headaches, seizures etc)? Or, is it only safe to assume that most do not cause problems? The bulbs in question are IKEA GA607 11W bulbs, but I doubt anyone knows the answer for that very model. ;) Aeluwas (talk) 18:33, 2 February 2008 (UTC)[reply]

Huh. Found this: "I have run a series of measurements on various electronically ballasted CFLs and found substantial amounts of 120 Hz flicker; typically 1/3 to 1/4 as much as found in linear lamps operated at 60 Hz on EM ballasts." link Aeluwas (talk) 18:54, 2 February 2008 (UTC)[reply]
I found that Compact fluorescent lamp caused interference on my infra red link headphones. I stopped using the headphones.--TreeSmiler (talk) 01:41, 3 February 2008 (UTC)[reply]
There are individual differences in the ability to see flicker. Some individuals have rejected CFLs on the ground that the flicker they see causes headaches. Temple Grandin has said that some autistic persons have a higher sensitivity to flicker than neurotypical individuals. If the electronic ballast in fact ran the fluorescent bulb at 20,000 Hertz, no one would see any flicker. 60 Hertz flicker might be perceptible to some people. Incandescent bulbs do not turn off completely during each AC cycle, because the filament cannot cool off that fast. Lower power incandescent bulbs have less thermal inertia and should show more flicker. Fluorescent bulbs are able to go on and off more completely during the low voltage part of each cycle. Edison (talk) 02:04, 3 February 2008 (UTC)[reply]
we have an article on this Flicker_fusion_threshold--TreeSmiler (talk) 02:17, 3 February 2008 (UTC)[reply]

how inefficient are wall warts?

I've read about how wall warts draw power whenever they're plugged in, but how much power? Does my 60W MacBook wart always dissipate 60W minus whatever is being used by the computer? If not, how does the power dissipated by the wart relate to the power used by the device? Thanks. --Allen (talk) 19:48, 2 February 2008 (UTC)[reply]

  • I am not sure about the Wall Warts, but I did hear, maybe on a recent "This Week in Science" podcast, that literally half our energy consumption is for things that are plugged in but not being used..things on "stand-by" so I imagine that the Wall Wart is also just drawing energy all the time rather the device connected to it needs energy or not. On that Wall Wart page it just says that 4% of energy use is attributed to such devices, but that is rather vague. Saudade7 20:43, 2 February 2008 (UTC)[reply]
    • I would expect a wall wart to draw substantially less than rated power when it is idling. That said, there was a time when "off" meant "off" for TVs and other appliances, with a single pole, single throw switch which killed all power flow. The multitide of plug in adapters are power vampires which draw some small amount of electricity 24/7 and have a very poor power factor. With a wattmeter you could measure the draw when the appliance is on versus off. If the appliance has a battery in it, like a laptop computer, then the idling draw should be greater than if the adapter is for an appliance which is switched off in the old-fashioned sense. A transformer has an "exciting current" which flows even if no power is drawn out of it.Edison (talk) 01:55, 3 February 2008 (UTC)[reply]

Teeth Whitening Strips

We are working on a multi-disciplinary experiment in chemistry/biology involving teeth whitening strips.

Each group used a different type of strips: Crest Classic, Listerine Dissolving, and Equate (Wal-Mart Brand). The active ingredients in all of these is hydrogen peroxide.

Could someone please help explain the science of how this works? Also, does anyone know the percent hydrogen peroxide in any of these strips or how to find this information? Any other additional information would be helpful also. —Preceding unsigned comment added by Kuanche (talkcontribs) 19:53, 2 February 2008 (UTC)[reply]

Hydrogen peroxide is an oxidiser, and works the same way bleach does to cloths. --antilivedT | C | G 21:32, 2 February 2008 (UTC)[reply]

ethylene

Does temperature affect the amount of ethylene produced from a fruit? —Preceding unsigned comment added by 70.79.131.252 (talk) 19:54, 2 February 2008 (UTC)[reply]

Yes. Higher temperature, more ethylene. [17]. --Allen (talk) 20:13, 2 February 2008 (UTC)[reply]

PHYSICS MAGAZINE QUESTION ABOUT AN ELEVATOR

Hey, it`s Me The Physics Magazine Guy I have this thing Here where two elevators are heading to the first floor of an office building. A scale is on the floor and a student is stepping on it. Another,is recording the speed as it moves up the elevator at rest. The elevator,goes up to the fifth floor. What,is the weight of the scale as it moves upward. The weight,the scale shows as velocity of the elevator becomes constant. The weight the scale shows as elevator slows down to a stop. The time the elevator took to make a trip from the 1st to the 5th floor. —Preceding unsigned comment added by 68.161.50.113 (talk) 19:57, 2 February 2008 (UTC)[reply]

You don't have enough information. -mattbuck 00:37, 3 February 2008 (UTC)[reply]
If the student weighed 150 pounds and the elevator took a very long time to go to the 5th floor, the scale would show 150 pounds. If it accelerated very swiftly, the scale might show 300 pounds or more. When it decelerated as it approached the target floor, it would show less than 150 pounds, even zero or negative pounds if the elevator were badly out of order, the scale was bolted to the elevator, and the student was strapped to the scale! Force equals mass times acceleration, in addition to the weight due to gravitational attraction. Your question is like asking what color the carpet in the hall was. Not enough information. Edison (talk) 01:49, 3 February 2008 (UTC)[reply]

Black Butterflies South American Plateau Fields of Crystals Art Bell...

I once heard an interview on Art Bell's late-night radio show Coast to Coast am. The guest was a man who was an explorer who was going to have a special on Discovery channel or History channel or another similar cable channel. He and his team had gone to some high mesa or butte or plateau in South America - maybe Peru or Brazil - to explore an area that was so geographically isolated from other ecosystems that the animals and plants there had evolved very differently. There were fields of white crystals and black rocks and butterflies were black as were various reptiles...maybe he even said there were black plants (???).

I think there was also a woman biologist in the expedition who worked on extremeophiles that lived in caves. She discovered that the stalactites in a cave there was actually some kind of colony - like a coral reef - of microorganisms that derived their nutrients from calcium-rich water seeping through the rock...something like that.

This has been driving me crazy for a long time. I have looked through all the "Coast to Coast am" past shows but none of the abstracts sound familiar. I feel like it was around summer of 06, but I am not sure. I have even Googled key words to no avail, but I do remember that right after the show aired there was a video from the show on the History Channel or Discovery Channel website so It really was a real expedition and not just a crazy person on Art Bell.

Any ideas of the explorers / scientists / place etc? Thanks Saudade7 20:36, 2 February 2008 (UTC)[reply]

Sounds like the table top mountains of Venezuela. Check out our article Tepui.--Eriastrum (talk) 21:05, 2 February 2008 (UTC)[reply]

Stone dropping falcon

While sailing we noticed a bird of prey - I believe a falcon - inspecting the boat from quite close. It stayed with us for about 5 minutes and before flying off dropped a 5 cm stone on us. What kind of behaviour would that be? Do Falcons take stones with them as ballast or as a tool for helping with balance. Or are they using it to stun a prey or test the objects or animals they encounter? Any suggestion welcome. Keria (talk) 21:57, 2 February 2008 (UTC)[reply]

I don't know about falcons dropping stones, but it is possible that the bird was a gull. They often will drop clams or other hardshelled animals on hard surfaces in order to break them open. They will sometimes drop stones by mistake.--Eriastrum (talk) 23:09, 2 February 2008 (UTC)[reply]
I'm not sure I know about any marine falcons. Ospreys catch fish; gulls and crows often drop hard food objects to break them. I tend to agree with Eriastrum. It may have been a gull. Bckirkup (talk) 23:23, 2 February 2008 (UTC)[reply]
This is interesting, but to be clear it was most definitely a raptor or bird of prey and not a gull. We sailed out of Freetown in Sierra Leone. These huge birds circle all day above the town. There are a lot of white belied crow like birds there too but this was much bigger than a crow and black with a short beak and with those long fretting feather at the tip of the wings. The mistaken clam idea sounds good though. Thank you. Keria (talk) 01:12, 3 February 2008 (UTC)[reply]
One is reminded of the unusual death of Aeschylus, killed when an eagle dropped a live tortoise on his (bald) head. The tortoise lived; Aeschylus - alas - not so much. - Nunh-huh 01:55, 3 February 2008 (UTC)[reply]

Tasting heat

I believe when breath starts forming into white condensation when exhaling it indicates temperatures of 8ºC and under (maybe varying with humidity in the air?). Is there an equivalent for high temperatures such as feeling the heat on ones palate or tong? Keria (talk) 22:01, 2 February 2008 (UTC)[reply]

Well the eight degree temperature that you quote, is being measured from a physical phenomenon, and has nothing to do with sensory perception from the mouth, so I don't understand your assumption of an "equivalent". If you put something in your mouth that's too hot, you notice, so you obviously have the ability to sense temperature via the mucosal tissues in your mouth. If you put something in your mouth at 8 degrees, you'll most likely notice that it's colder than an object of similar composition at 20 degrees. Likewise, you'll notice a difference between something at 39 degrees compared with 60 degrees - there's no special single temperature that could be deemed in any way "equivalent" to the temperature at which moisture in exhaled breath condenses. ----Seans Potato Business 23:43, 2 February 2008 (UTC)[reply]
Ah ok, let's make it this then: What ways are there to estimate the temperature in hot climates without the use of a thermometer? Keria (talk) 01:15, 3 February 2008 (UTC)[reply]
If you can fry eggs on your belly, its hot!--TreeSmiler (talk) 01:48, 3 February 2008 (UTC)[reply]
Bimetalic strips? You could have several configured for indicating different temperatures... maybe you'd only need one and can calibrate it against a real thermometer so that the temperature reading is given by the degree of bending. ----Seans Potato Business 02:07, 3 February 2008 (UTC)[reply]
Oh, apparently that might count as a thermometer... does it have to be sensed with your body? Just touch it and guestimate? --Seans Potato Business 02:10, 3 February 2008 (UTC)[reply]

Ants on the dole

An entomologist friend told me that amongst ants and bees there where individuals who wouldn't fit their usual caricature as diligent working insects but would do nothing to help the colony and profit of the work of others. Is this true and has it been studied? Keria (talk) 22:04, 2 February 2008 (UTC)[reply]

There are several kinds of phenomena that fit the general description you suggest. Noe and Hammerstein discuss the general phenomena in their article "Biological Markets" Trends in Ecology & Evolution; Volume 10, Issue 8, August 1995, Pages 336-339. Among ants there are various forms of social parasitism and "cheating." In ant-plant symbioses there ants who "cheat" the plants. Studies are numerous. Perhaps start with Itino et al, Ecological Research; Volume 16, Number 4; December, 2001. It studies part of the problem. Social parasitism describes certain species of ant that profit from others (somewhat like birds that use other birds to incubate their eggs; like the cuckoo - as opposed to nest stealing, like grosbeaks). I recommend Annual Review of Entomology 2001, Vol. 46: 573-599 - Lenoir et al, "Chemical Ecology and Social Parasitism in Ants" to get some detail as to how this works. Some ant colonies may contain lazy workers - but this is a problem for the colony. Mo re often the problem is not laziness but illicit egg-laying. There are ways that ants (and bees or wasps) deal with such problems in the colony. You might read D'Ettorre 2004 (Proceedings of the Royal Society of London B) to learn about this. People are starting, perhaps, to move away from studying such pairwise interactions as a mutualism with cheaters, which will make the whole topic more complex. Try Stanton, 2003 (American Naturalist vol. 162, no4, pp. S10-S23) for a review of this change. Bckirkup (talk) 23:19, 2 February 2008 (UTC)[reply]

Speed of waves

I think waves all propagate at the same speed in water. Is this true and if so what is this speed? How do temperature and salinity influence it? Keria (talk) 22:14, 2 February 2008 (UTC)[reply]

Longitudinal waves and transverse waves can travel at different velocities in the same medium. These would correspond to underwater acoustics and ocean surface waves, respectively. The speed of each is explained in underwater acoustics#Speed of sound, density and impedance and ocean surface waves#Science of waves, although I'll admit the latter is not very satisfying. Someguy1221 (talk) 22:53, 2 February 2008 (UTC)[reply]
And specifically, if you follow the last link above (which I've fixed), you'll see that the answer is no. Roughly speaking, bigger waves travel faster; tsunamis are the fastest of all. --Anonymous, 23:30 UTC, 2008-02-02.
Oh yes, forgot. As with the above, the velocity of a water wave varries with its wavelength/frequency. Someguy1221 (talk) 00:20, 3 February 2008 (UTC)[reply]

Zen monks pain suppression

I've heard several times that some trained shamans or monks can suppress the feeling of pain. The latest mention of it was in the SciFi film '[The man from earth]' (which I recommend). Is this pure fiction or has it a basis in reality? How would it work? Would it be a real suppression or 'only' a great tolerance to pain? Thank you. Keria (talk) 22:21, 2 February 2008 (UTC)[reply]

Given that Thich Quang Duc peformed self-immolation in front of a crowd and was photographed, and that a few other monks have done the same or similar...I'd say there's gotta be some truth to it. Beyond that, I have no idea. It would have to be either stopping the signals from reaching the brain or just ignoring them once they got there...but that's obvious. I suppose, if they're in a trance or whatever, that some parts of their brains have shut down (sorta like when you sleep), and if one of those parts is the part that processes pain, they won't feel it. Of course, you'll feel pain even when you're asleep if it's strong enough, so who knows how they would do that...there have been a few studys into it (I think), but I don't know where they are. Trimethylxanthine (talk) 23:17, 2 February 2008 (UTC)[reply]
Having had surgery while conscious, I can attest that medications make it possible to still feel the pain, but to not be troubled by it. Perhaps a monk would be able to transcend dental medication while getting a tooth cavity drilled out. Edison (talk) 01:42, 3 February 2008 (UTC)[reply]

Regulatory region vs sequence

"Regulatory region" gets more google hits than "regulatory sequence". Can I confirm a preference for regions (for merging purposes (gathering consensus on the talkpages will take months)). ----Seans Potato Business 23:14, 2 February 2008 (UTC)[reply]

I get 373 hits for "regulatory sequence" and only 254 for "regulatory region". It says 159,000 and 338,000 hits respectively on the first page, but if you actually go through the pages (repeatedly clicking on the rightmost "o"), they end after 373 and 254 respectively. I think it's a really bad idea to use Google hit counts as evidence of anything. -- BenRG (talk) 01:33, 3 February 2008 (UTC)[reply]
That's outrageous! I knew they were up to something and now I've got all the proof I need!! Thanks :) ----Seans Potato Business 01:59, 3 February 2008 (UTC)[reply]

Physics pendulum question

What would be other examples where a pendulum would be used for timing? I already thought of a metronome and a clock, is anyone able to think of anything else? Icestorm815 (talk) 23:44, 2 February 2008 (UTC)[reply]

Surely using a pendulum as timing is always just a variation on a metronome. -mattbuck 00:33, 3 February 2008 (UTC)[reply]

Video

Watch this video then try and answer my questions

1) Does his theory on how to cure cancer even make sense?

2) Is there alternate fuel even sensible? Will it ever be put to use? Does it use too much salt water to work well?

Thanks, schyler (talk) 00:43, 3 February 2008 (UTC)[reply]

1) There is Radiofrequency ablation, but he'd need a way to make his metal particles selectively enter cancer cells, which the video never explains.
2) Absolute bullshit. Saltwater is, chemically speaking, fantasitcally stable. The process of burning would necessarily consume more energy in the form of radio waves than it releases as the products are burned. At best, this is just an efficient way of turning electrical energy into mechanical energy, but I'm not sure of that. Someguy1221 (talk) 01:05, 3 February 2008 (UTC)[reply]

Conveying thought process in audio/visual context

On television etc, the idea that someone is thinking is conveyed by changing the sound of their voice-I think it gets a bit deeper and echoey. Exactly what parameters are changed to achieve this effect and how might I go about emulating them in Windows and/or Linux?

Magnetic field

There is an ideal n × n × n lattice of aligned magnetic dipoles, d units apart. How much energy would be required to rotate the central dipole 180°, in terms of n, d, and any other nessecary variables? Thanks *Max* (talk) 02:10, 3 February 2008 (UTC).[reply]